Download as pdf or txt
Download as pdf or txt
You are on page 1of 74

SESSION 1

READING STRATEGIES

I – GUESS THE MEANING FROM CONTEXT


In the reading section of the test, you may be asked to find out the meaning of a word or
phrase in the text. That word or phrase may be the difficult one that you have never seen
before. They may also have different meanings. However, the text will provide you an
explanation or example of the meaning of that word or phrase. What you need to do is to
look at those examples or explanations in the text. This context will give you the clue to
understand the word or phrase.
Ex:
Some of the trees in Redwood National Park are truly gargantuan, and in fact they are the
tallest trees in the world.
In this context, the word ‘gargantuan’ means ‘very large’.
There are four common types of context clues:
1. Examples
Signals of examples in the text: for example, for instance, such as, including, like, as an
example, cases of, instances of, type of, (e.g)…
Ex:
Although the hazards of the trip were many – for example, the unbearable heat, the lack of
water, the possibility of getting lost, the presence of wild animals and poisonous snakes –
the man nevertheless decided that he must go.
The examples listed in the text can help you understand the meaning of the word ‘hazards’,
which means ‘dangers’.
2. Synonyms
Signals: means, called, that is, also known as, or, is referred to as, is/are known as, by…is
meant…, similar to, can be defined as; or the phrases following the comma (,) dash ( - ), or
parentheses ( )…
Ex:
Mark wanted to impress his date with the dinner he prepared, but the massive ice sculpture
centerpiece he made with a chainsaw between courses was just superfluous, extra and
unnecessary.
In this context, there are two synonyms of the word ‘superfluous’ given. And that’s also the
meaning of the word: extra and unnecessary.
3. Antonyms
Signals of antonyms in the text: however, but, yet, in contrast, although, on the other hand,
despite, while, in spite of, even though, unlike, nevertheless…
Ex:
It kept me dry and warm even though I was soaked, and all of the moisture on my body
passed right through.
The word ‘soaked’ here is somehow opposite to the phrase ‘dry and warm’; therefore, its
meaning can be ‘wet’.
1
4. General sense of the sentence or text
Readers need to rely on their experience and background knowledge to figure out the
meaning of the word.
Ex:
The strong wind will make your hair appear scraggy.
In this context, the word ‘scraggy’ means ‘untidy.’
Sometimes, the readers can guess the meaning of the word based on its root, the prefix or
suffix. Following are some common roots, prefixes and their meanings.
ROOTS/
MEANING EXAMPLES
PREFIXES
A, an Not, without Anonymous, apathy, atheist
Anti, ant Against, opposite Antisocial, antibody, antiseptic
Audi To hear Audience, audible, audition
Auto Self Automobile, automatic, autograph
Chron Time Chronology, chronic, chronicle
E, ex Out, away, from Emit, exhale, exceed
Hyper Over, above Hypertensive, hypersensitive, hyperactive
In, im Not Innocent, impossible, inviolate
Intra Within, into Intranet, intracranial, intravenous
Mal Bad, badly Malcontent, maleficent, dismal
Meta Beyond, change Metaphor, metabolism, metachromatic
Omni All Omnipotent, omnipresence, omniscient
Port To carry Portable, deport, import
Spect To look Inspect, spectator, prospect
Distance, from
Tele Television, telegraph, telemetry
afar
Vita Life Vital, vitamins, revitalize

Practice 1
What does the bold word in each sentence mean? Choose the best answer.
1. After the harvest, we had an abundant amount of apples. We made apple pie, apple
sauce, and apple juice because we had so many apples.
A. a shortage C. not enough
B. very red D. plentiful
2. When Sara was sick, her voice was almost inaudible. We couldn’t hear what she
was trying to say clearly.
A. very loud, easy to hear C. very shy, introverted
B. very strange, uncommon D. very soft, hard to hear
2
3. Petra has so many friends because she is a gregarious person.
A. introverted, self-contained C. shy, quiet
B. rude, hostile D. friendly, outgoing
4. Most domestic dogs are smart. They can be trained for hunting. They can also be
taught to be watchdogs or guide-dogs for blind people.
A. wild C. hunting
B. overseas D. tame
5. Because of the warm, moist climate, almost any crop can be grown here, including
rice, cotton and sugar cane. Tobacco and even bamboo grow well. This city contains
one of the largest sugar refining plants in the world.
A. natural substances C. fruit-trees
B. forest trees D. agricultural plants
6. Bangkok has many important historic shrines. These include the temple of
the Reclining Buddha, and the Equestrian Statue of Rama V.
A. Places with many government agencies and bureaus
B. Places in which objects illustrating art, history, sciences, etc. are displayed
C. Places on a sports ground for use of players, spectators, etc.
D. Places associated with something or somebody deeply respected
7. The life spans of different rodents vary greatly. For example, a rabbit has a
life span more than twice as long as that of a rat and almost four times as
long as that of a mouse.
A. Animals with six legs and no backbones
B. Animals which feed their young with milk
C. Animals which are kept in a zoo
D. Animals with teeth adapted for biting
8. Nocturnal creatures, such as bats and owls, have highly developed senses
that enable them to function in the dark.
A. Feathery C. Dark
B. Living D. Active at night
9. While Mecca’s apartment is decorated plainly, her clothing is very
flamboyant.
A. inexpensive C. flowery
B. washable D. flashy
10. Cockroaches don’t eat much, but they spoil food with their body wastes. They
carry germs of diseases, including polio and typhoid fever that affect humans.
A. supply C. chew
B. make D. damage

3
Practice 2
Land subsidence is the downward shifting of the ground. The gradual sinking of the
earth can result in millions of dollars worth of damages to roads and property.
According to some reports, every state in the United States suffers from land
subsidence to some degree. Although sometimes land can sink because of natural
causes, the two most common reasons for land subsidence are caused by humans.
Humans encourage land subsidence by pumping groundwater. According to some
estimates, 80 percent of all land subsidence in the world is caused by the extraction
of water from underground sources, such as aquifers. When the water is pumped
from under the surface of the earth, the ground above the well becomes weak. The
loss of support from the water beneath it also causes the soil to become more
compact. Thus, the soil takes up less space. Since the volume of the soil decreases,
the ground shifts downward. In Mexico City, for example, the pumping of
groundwater has had a dramatic effect on the ground level. Pumping has caused
the ground to sink at least thirty feet over the past century.
Certain types of mining also cause land subsidence. Mining is the practice of digging
into the earth in order to extract minerals or metals. Before miners can reach the
minerals, they must dig long shafts. In the process of digging the shafts, large chunks
of ground are removed from the Earth. The shafts tunnel deep into the ground. For
example, the longest mine shafts in the world are over two miles long. The creation
of mines leaves large, empty pockets of space underground. Eventually, the ground
above the pockets is not able to support itself. Engineers can help support the
ground by refilling mines with concrete or sand after it is used. However, sometimes,
the tunnels are simply abandoned. Without proper design, mining induces sinking
in the ground directly over the area where the mine is built. It causes the surface to
tilt and curve. This can be very costly if it affects buildings or infrastructure.
1. The word gradual in the passage is closest in meaning to
A. small C. slow
B. dramatic D. downward
2. The word compact in the passage is closest in meaning to
A. weak C. heavy
B. packed D. wet
3. The word induces in the passage is closest in meaning to
A. checks C. reduces
B. stops D. generates
Practice 3
Hellenism refers to the ideals, rituals, and life of ancient Greece, primarily referring
to the culture of Athens. The Hellenistic culture was a blend of Greek, Egyptian,
Indian, and Persian cultures. In 334 BC, Alexander the Great invaded and
conquered the Persian Empire. After that, he could spread Greek Culture to the
conquered lands. Even after his death, Greek culture continued to proliferate, and
law, politics, art, philosophy, literature, and religion were all exported. In this way,
Greek culture was spread to other civilization and cultures, and Hellenistic
civilization became an integral part in Western Civilization.

7
Due to Alexander’s conquest, many Greeks left home to work in foreign countries.
The Greek countryside was completely abandoned; foreigners and slaves took the
place of those gone. To entice Greeks to return, citizens used their wealth for public
display, rather than private consumption. The urban and upper class character of
Greece spread to the upper classes abroad. The Greek style of life strongly
appealed to the masses.
1. The word proliferate in the passage is closest in meaning to
A. emerge C. deteriorate
B. propagate D. mingle
2. The word entice in the passage is closest in meaning to
A. convert C. assure
B. deceive D. lure

II – RECOGNIZE REFERENTS
You may be asked to determine the referent for a particular pronoun or adjective; or
find the referent for a variety of words. A referent generally precedes the pronoun or
adjective in the passage; so, to answer this question type, you should study the
context around that.
Ex:
A tornado is created when warm, moist air rises from the ground and comes into
contact with a mass of colder air at the bottom of a thundercloud. The rising air
pushes against the colder air, and the rotation of the air to spin, in much the same
way that water in a sink spins as it goes down a grain.
The pressure at the center of a tornado is much lower than that in the air surrounding
the tornado. The low pressure creates a funnel in the middle of the tornado, which
causes destruction by acting much like a vacuum cleaner and sucking up whatever
is in its path.
The word it in paragraph 1 refers to
A. way
B. water
C. sink
D. drain
Practice 1: Horatio Alger, Jr. (1832 – 1899) was the author of more than 100 books
for boys in the second half of the nineteenth century that focused on the theme of
success coming to those who work hard to achieve it. The son of a minister, Alger
came from a prominent Massachusetts family. He graduated with honors from
Harvard in 1852
and graduated from the Cambridge Divinity School eight years later. He served as
a minister for a short time before moving to New York City in 1866 to devote his time
to writing inspirational books for boys.
In many of his books, he wrote about the poor and homeless children of the slums
of New York City, seeing them as unfortunate pawns of society who, if only given

2
the opportunity, could improve their lot. A general plotline that he followed often was
of a poor boy who managed to achieve a respectable and successful life by working
hard and taking advantage of opportunities presented. Though his writing style was
characterized by simplicity and repetition, it was well received by his target audience;
his books were enormously popular, selling millions of copies well into the first few
decades of the twentieth century.
1. The word that in paragraph 1 refers to
A. author C. boys
B. books D. half
2. The word it in paragraph 1 refers to
A. The second half C. 100
B. The nineteenth century D. success
3. The word who in paragraph 2 refers to
A. slums C. pawns
B. society D. opportunity
Practice 2
Iron (Fe on the periodic table) is created through a long process that takes place in
the center of stars. When a star reaches the end of its life cycle, it explodes, and the
iron is released. The metal can be found in places all over the Earth, but scientists
aren’t sure exactly how it got here. They have come up with two separate theories
to explain how iron showed up on the planet.
People from various ancient cultures, including the Egyptians and Greeks, believed
that iron fell from the heavens in the form of meteorites. In fact, one ancient word for
iron, ‘anbar’, means ‘fire and sky’. Some scientists agree with this early theory. They
have found that iron is a material in some of the meteorites found on Earth. However,
meteoric iron is rare and is mostly found in museums today.
Others think that iron was deposited onto the planet while it was still forming. Billions
of years ago, a neighboring star exploded and shot out hot gases onto it. The gases
turned into iron when they cooled. As the Earth continues to form, the iron sunk to
the center of the planet. Scientists explain that, as a result, large amounts of it are
found in the Earth’s core.
1. The word it in paragraph 1 refers to
A. star C. metal
B. life cycle D. earth
2. The word others in the passage refers to
A. Greeks C. Meteorites
B. Scientists D. Museums
3. The word it in paragraph 3 refers to
A. Earth C. center
B. Iron D. result

10
III – FIND FACTUAL INFORMATION
When you are asked questions about factual information, the answers to these
multiple choice questions are often restatements of what is given in the passage.
The following chart outlines the key information that you should remember about
questions testing details
QUESTIONS ABOUT FACTUAL DETAILS
How to identify the According to paragraph X…
question It is stated in paragraph X…
It is indicated in paragraph X….
It is mentioned in paragraph X….
Where to find the The answers are generally found in order in the passage,
answer and the paragraph where the answer is found is generally
indicated in the question.
How to answer the 1. Choose a key word or idea in the question.
question 2. Skim the appropriate paragraph for the key word or idea.
3. Read the sentence that contains the key word or idea
carefully.
4. Eliminate any definitely wrong answers, and choose the
best answer from the remaining choices.
Ex:
John went to the store and bought apples. After that, he went to the bank before
stopping at the post office to mail some letters. Back home, he just watched some
TV and took a nap.
1. What did John buy?
A. Apples C. Bananas
B. Oranges D. Carrots
2. Why did John go to the post office?
A. to mail some letters
B. to buy stamps
C. to talk to his friend
D. to get a package

1
Practice 1
Scientists have long debated the way that humans first colonized the Americas. It is widely
believed that humans crossed the Pacific Ocean from Asia.
Scientists support this theory with anthropological evidence. To find similarities between
the two cultures, scientists have studied teeth. Because teeth remain for a very long time,
they provide accurate records of the past. Researchers have found that the structure of
Native Americans’ teeth is more similar to those of Asians than any other culture in the
world. One study of over 15000 cranial remains found that the teeth of Native Americans
and Asians fell into a group of similar tooth patterns called sinodonty.
In addition, scientists have found archaeological evidence to support the theory. This
evidence comes in the form of human remains found in a cave in Southeast Alaska.
After researchers analyzed the remains, they found that he mainly survived on a diet of
marine life such as fish. Fish, however, was not a diet that would have been readily
available in that location. During that period of history, many coastal Asian cultures had
diets of mainly marine life. Thus, the man only would have had access to fish if he had
come from Asia.
1. Why do teeth provide accurate information about past cultures?
A. They retain DNA evidence after a person has died.
B. They remain long after a body has decayed.
C. They show traits unique to different individuals.
D. They can be used to determine a person’s diet.
2. According to paragraph 2, sinodonty is
A. A group of cultures with similar teeth.
B. The study of cranial and dental remains
C. A category used to describe dental remains
D. A method of analyzing the remains of teeth.
3. What have scientists learned from the remains of a human found in Southeast
Alaska?
A. He survived on marine life found nearby.
B. He died from a lack of food.
C. He could not properly digest fish.
D. He ate a diet common to Asia.

1
Practice 2
Crescent-shaped Lake Baikal, in Siberia, is only the ninth largest lake in area at 385 miles
(620km) in length and 46 miles (74km) in width, yet it is easily the largest body of the total
of all the water in the five Great Lakes; it holds so much fresh water in spite of its less-
than-impressive area because it is by far the world’s deepest lake. The average depth of
the lake is 1312 feet (400 meters) below sea level, and the Olkhon Crevice, the lowest
known point, is more than 5250 feet (1600 meters) deep.
Lake Baikal, which today is located near the center of the Asian peninsula, is most likely
the world’s oldest lake. It began forming 25 million years ago as Asia started splitting
apart in a series of great faults. The Baikal Valley dropped away, eventually filling with
water and creating the deepest of the world’s lakes.
1. What is stated in paragraph 1 about the shape of Lake Baikal?
A. It is wider than it is long.
B. It is circular in shape.
C. Its width is one-half of its length.
D. It is shaped like a new moon.
2. It is indicated in paragraph1 that the area of Lake Baikal
A. Is less than the area of eight other lakes
B. Is one-ninth the area of Siberia
C. Is greater than the area of any other freshwater lake
D. Is equal to the area of the five Great Lakes
3. According to paragraph 1, the Olkhon Crevice is
A. Outside of Lake Baikal
B. 400meters below sea level
C. The deepest part of Lake Baikal
D. 5000 meters deep
4. It is mentioned in paragraph 2 that Lake Baikal
A. Is not as old as some other lakes
B. Formed when sections of the earth were moving away from each other
C. Was fully formed 25 million years ago
D. Is today located of the edge of the Asian peninsula

2
IV – UNDERSTAND NEGATIVE FACTS
In the reading test, you will sometimes be asked to find an answer that is not stated, or
not mentioned, or not true in the passage. This type of question really means that three
of the answers are stated, mentioned or true in the passage, while one is not. There are
two kinds of answers to this type of question:
- There are three answers that are true and one that is not true according to the
passage.
- There are three true answers and one that is not stated or mentioned in the
passage.
The following chart outlines the key information that you should remember about
questions testing negative facts
QUESTIONS ABOUT NEGATIVE FACTS
How to identify the It is NOT stated…
question It is NOT mentioned…
It is NOT true…
It is NOT indicated…
All of the following are true EXCEPT…
Where to find the These answers are generally found in order in the passage,
answer and the paragraph where the answer is found is generally
indicated in the question.
How to answer the 1. Choose a key word in the question.
question 2. Scan the appropriate place in the passage for the key word
(or related idea).
3. Read the sentence that contains the key word carefully.
4. Look for the answers that are definitely true according to
the passage. Eliminate those answers.
5. Choose the answer that is not true or not discussed in the
passage.

Ex: Many people are quite familiar with rainbows, but few are as familiar with moonbows.
Rainbows are caused by sunlight hitting raindrops and bouncing back. You can see a
rainbow when the Sun is low in the sky behind you and it is raining ahead of you. Light
from the Sun reflects off the inside the surfaces of raindrops and is bent as it travels
through them. It appears as a band of colors because each of the colors in sunlight is
bent to a different angle.
Moonbows are far less common than are rainbows, but they are formed in much the same
way. They require a very specific set of circumstances to occur. When they do occur, they
occur just after a full Moon, a Moon at its brightest, has risen in the east and just after the

3
Sun has set in the west, and it must also be raining in the west. In this situation, a
moonbow may be visible to you if you are facing west and if the Moon is behind you. Light
from the bright Moon reflects off the inside surfaces of the raindrops in the west and bends
the colors to create a moonbow.
It is NOT indicated in the paragraph 2
A. where the moon must be in the sky for a moonbow to occur
B. at what time of day moonbows occur
C. which direction you must be facing to see a moonbow
D. in which parts of the world moonbows occur
Practice 1
The Mesner Acting Technique was created by Sanford Meisner in the mid-1900s. It does
not focus on reading scripts. Instead, the Meisner Technique teaches students how to
spontaneously act according to a situation. The technique is now taught in several acting
schools and has been used to train several Hollywood stars.
The Meisner Technique begins by focusing on acting without planning. The actors are
given a prompt and must move around as if they were in that setting. An actor might be
told to act as if he were exploring the jungle. The actor then must move on stage without
a script, wiping away sweat, swatting at bugs, etc. in this way the Meisner technique
teaches actors to place themselves into the roles they are playing and move in a realistic
way.
Next, the Meisner technique teaches emotional speech. Actors are given a prompt and
must interact and talk, while showing emotions that they would show in a ‘real life’
situation. Since actors do not have a script to refer to, they have to imagine themselves
in a similar emotional setting. For example, an actor would need to first analyze a situation
and then react in a way that he or she normally would. Thus, the actor’s lines are both
more realistic and more emotional.

1. All of the following are elements of the Meisner Technique EXCEPT:


A. spontaneous acting
B. acting without a script
C. moving without planning
D. exaggerating emotions
2. All of the following are true of the Meisner Technique EXCEPT
A. actors create their own lines
B. actors are often given a prompt
C. actors begin by evaluating the situation
D. actors study the performances of the actors

4
Practice 2
Members of the flatfish family, sand dabs and flounders, have an evolutionary
advantage over many colorfully decorated ocean neighbors in that they are able to
adapt their body coloration to different environments. These aquatic chameleons have
flattened bodies that are well-suited to life along the ocean floor in the shallower areas
of the continental shelf that they inhabit. They also have remarkably sensitive color
vision that registers the subtlest gradations on the sea bottom and in the sea life around
them. Information about the coloration of the environment is carried through the nervous
system to chromatophores, which are pigment-carrying skin cells. These
chromatophores are able to accurately reproduce not only the colors but also the
texture of the ocean floor. Each time that a sanddab or flounder finds itself in a new
environment; the pattern on the body of the fish adapts to fit in with the color and texture
around it.

1. It is NOT stated in the passage that sand dabs


A. are a type of flatfish
B. are in the same family as flounders
C. have evolved
D. are colorfully decorated
2. According to the passages, it is NOT true that sand dabs and flounders
A. have flattened bodies
B. live along the ocean floor
C. live in the deepest part of the ocean
D. live along the continental shelf
3. All of the following are stated about the vision of sand dabs and flounders EXCEPT
that they are
A. overly sensitive to light
B. able to see colors
C. able to see the sea bottom
D. aware of their surroundings
4. It is NOT true that chromatophores
A. are skin cells
B. carry pigment
C. adapt to surrounding colors
D. change the ocean floor

5
5. It is NOT mentioned in the passage that sand dabs and flounders
A. move to new environments
B. adapt their behavior
C. can change color
D. adapt to textures around them

V – SIMPLIFY MEANINGS OF SENTENCES


With this type of question, you must choose the one answer that is closest to the meaning
of a sentence that is highlighted/ bold in the passage. To answer the question, you should
break the heighted sentence down into meaningful parts.
The following chart outlines the key information that you should remember about
questions testing the similar meaning of sentences.

QUESTIONS ABOUT SIMPLIFYING THE MEANINGS OF SENTENCES


How to identify the Which of the sentences below best expresses in the essential
question information….?
Where to find the The targeted sentence is highlighted or bold in the passage.
answer Information to answer the question is in the bold sentence
and may also be in a context around the bold sentence.
How to answer the 1. Study the bold sentence carefully.
question 2. Break the sentence down into meaningful parts by looking
for punctuation and transition expressions.
3. If the bold sentence makes references to information
outside of the bold sentence, read the context around the
bold sentence.
4. Study the answer choice, and eliminated definitely
references.
5. Choose the best answer from the remaining choices.
Ex: Of all sports injuries, the most common are due to damage to soft tissue. Such injuries
often involve the knees, shoulders, elbows, and wrists. Many times, a small injury to soft
tissue may make an athlete compensate by altering his or her normal movements
to protect the injured area. This can cause trouble because the change in normal
movement may put a sudden strain on uninjured areas, resulting in further injuries for the
athlete. In this way, one injury will often lead to another, with each new injury becoming
progressively more serious. Kinesiologists, or coaches with special knowledge of
kinesiology, can predict the complications that may arise from altering normal
movements and thus avoid muscle strains, torn rotator cuffs, ligament damage,
and herniated tissue.

6
Which of the sentences below express the essential information in the bold sentence in
passage? Incorrect choices change the meaning in important ways or leave out essential
information.
A. Kinesiologists can solve the problems caused by muscle injuries because they have
extensive medical training in repairing tissue.
B. Coaches who understand kinesiology can prevent injuries because they understand
how altering muscular motion can cause problems.
C. Before any serious injuries can occur, coaches should consult a kinesiologist,
someone who can help an athlete avoid muscle-related problems.
D. Kinesiologists have a special knowledge of how muscle movements will affect an
athlete’s chances of suffering muscular damage.
Practice 1: Camouflage is one of the most effective ways for animals to avoid attack in
the treeless Arctic. However, the summer and winter landscapes there are so diverse
that single protective coloring scheme would, of course, prove ineffective in one
season or the other. Thus, many of the inhabitants of the Arctic tundra change their
camouflage twice a year. The arctic fox is a clear-cut example of this phenomenon;
it sports a brownish gray cote in the summer which then turns white as cold
weather sets in, and the process reverses itself in the springtime. Its brownish-gray
coat blends in with the barren tundra landscape in the months without snow, and the white
coat naturally blends in with the landscape of the frozen wintertime tundra.
1. Which of the sentences below express the essential information in the first bold
sentence in the passage? Incorrect choices change the meaning in important ways or
leave out essential information.
A. Opposite conditions in summer and in winter necessitate different protective
coloration for Arctic animals.
B. The coloration of the summer and winter landscapes in the Artic fails to protect the
Arctic tundra.
C. In a single season, protective coloring schemes are ineffective in the treeless Arctic.
D. For many animals, a single protective coloring scheme effectively protects them
during summer and winter months.
2. Which of the sentences below express the essential information in the second bold
sentence in the passage? Incorrect choices change the meaning in important ways or
leave out essential information.
A. The arctic fox is unusual in that the color of its coat changes for no reason.
B. The arctic fox lives in an environment that is brownish gray in the summer and white
in the winter.
C. It is a phenomenon that the coat of the arctic fox turns white in the springtime and
gray in the fall.
D. The arctic fox demonstrates that protective coloration can change during different
seasons.

7
Practice 2
The size of many cities is increasing. In many places, the growth of cities has meant
the destruction of forests; however, in others, planners have recognized the
importance of maintaining nature. Many cities are home to urban forests, or collections
of trees that grow within a city or near heavily populated areas. Urban forests provide a
number of benefits.
Scientists have learned that urban forests can help reduce pollution in cities. Through a
natural process, the leaves of a tree absorb harmful gases from the atmosphere.
According to one study, a single tree can absorb about thirteen tons of carbon dioxide per
year. Scientists have also found that trees are capable of removing many other
pollutants that are believed to be responsible for global warming, such as carbon
monoxide and ozone.
Urban forests are also beneficial to cities because they help regulate temperatures. The
concrete used to construct buildings and sidewalks in cities soak up heat from the sun. it
can be up to nine degrees warmer in cities than in nearby rural areas. However, the trees
help cool down urban areas by providing shade. During the winter, buildings are shielded
from frigid blasts of air by trees. As a result, buildings do not become as cold.

1. Which of the sentences below express the essential information in the bold sentence
in paragraph 1? Incorrect choices change the meaning in important ways or leave out
essential information.
A. City planners can help reduce the destruction of forests by discouraging the growth
of cities.
B. Many city planners understand that nature is often harmed by construction within
cities.
C. Nature is recognized as an important factor in the growth and maintenance of cities.
D. Destruction of forests can be stopped in cities by city planners.
2. Which of the sentences below express the essential information in the bold sentence
in paragraph 2? Incorrect choices change the meaning in important ways or leave out
essential information.
A. Trees can affect the levels of carbon monoxide in an area, leading to global
warming.
B. Trees are able to remove carbon monoxide and ozone from the atmospheres but are
responsible for global warming.
C. Scientists think that trees can be used to find a link between pollutants and global
warming.
D. Trees can be used to reduce global warming by reducing harmful pollutants from
the atmosphere.

8
VI – MAKE INFERENCES
You may be asked to answer a multiple choice question by drawing a conclusion from a
specific detail or details in the passage. This type of questions contain the words implied,
inferred, likely, or probably to let you know that the answer of the question is not directly
stated. Therefore, you need to draw a conclusion from the information of the passage.
The following chart outlines the key information that you should remember about
questions testing inferences.
QUESTIONS ABOUT INFERENCES FROM STATED FACTS
How to identify It is implied in paragraph X…
the question It can be inferred from paragraph X…
It is most likely that….
What probably happened …?
Where to find The answers to these questions are generally found in order in
the answer the passage
How to answer 1. Choose a key word or phrase in the question.
the question 2. Scan the passage for the key word or phrase
3. Carefully read the sentence that contains the key
Ex:
Although the use of candles can be traced back to about 400AD, they did not become
widely used until the 14th century. The best candles were made of beeswax, but these
were generally confined to use in religious rituals in churches. Most average people of
the time preferred to use inexpensive tallow candles made from animal fat. Tallow candles
may have been cheap, but they tended to be smoky and smelly, and dripped profusely
while giving very weak light.
In the paragraph, the author suggests that
A. beeswax candles were too expensive for everyday use.
B. there were religious rules against tallow candles.
C. most people did not mind the smell of tallow candles.
D. tallow candles did not have good points

9
Practice 1
Aroma therapy uses scents from many plants to aid both the body and the mind. For
years, scientists have debated the scientific merit of aromatherapy. Recently, a number
of studies have shown that aromatherapy does have medical benefits.
Many of the substances – called essential oils – used to produce scents have been shown
in studies to affect the body. Perhaps the most surprising find was the ability of a number
of oils to fight viruses. The herpes simplex virus is common in humans and can lead to
infections, sores, and even some forms of cancer. A number of oils, including sandalwood
oil and peppermint oil, were found to be effective in fighting the virus. Scientists believe
that chemicals within the oils can destroy the cells of the virus without harming healthy
cells.
More commonly, essential oils are used to help those with emotional problems. Scientists
have begun research into aromatherapy’s uses in anxiety disorders. In one study, mice
were placed into high-stress situations. The scientists monitored their anxiety levels.
Scientists released lemon oil vapors and reviewed the mice’s anxiety levels. They found
that lemon oil reduced the anxiety levels of the mice. The oil targets a specific area in the
brain that is usually responsible for increased stress levels.
1. According to paragraph 1, which of the following can be inferred about aromatherapy’s
history?
A. Some scientists in history have doubted its benefits.
B. It has been in use longer than traditional medicine.
C. It had not been researched until the 20th century.
D. Ancient societies widely used aromatherapy.
2. According to paragraph 2, what can be inferred about the herpes simplex virus?
A. It is a dangerous risk to most people’s health.
B. It is more harmful than once thought.
C. It does not affect people who use aromatherapy.
D. Scientists aren’t certain how essential oils affect it.
3. What does the author imply about the mice used to test the effects of aromatherapy?
A. They were selected because they had naturally high anxiety.
B. Essential oils were responsible for changes in their brain chemistry.
C. Only mice that had been exposed to lemons responded to the therapy.
D. Their brains did not contain the receptor that controls stress.

10
Practice 2
One of the most beautiful of the more than 100,000 known species in the order
Lepidoptera are the tiger moths, moths known for the striking appeal of their distinctive
yellow-and-black patterns of spots and stripes. Such boldly patterned color combination
is commonplace in the animal world, serving the function of forewarning potential
predators of unpleasant tastes and smells. This is unquestionably the function served by
the striking coloration of the garden tiger moth, which is quite visually attractive but is also
poisonous to predators. Certain glands in the garden tiger moth produce strong toxins
that circulate throughout the insect’s bloodstream, while other glands secrete bubbles that
produce a noxious warning smell. The tiger moth, indeed, is a clear example of a concept
that many predators intuitively understand; that creatures with the brightest coloration are
often the least suitable to eat.
1. It is implied in the passage about the order Lepiodoptera that
A. All members ò the order are moths
B. there may be more than 100,000 species in this order
C. all members of the order are brightly colored
D. there are most likely fewer than 100,000 species in this order

2. It can be inferred from the passage that the tiger moth was so named because
A. Its coloration resembles that of a tiger
B. It is a ferocious predator, like the tiger.
C. Its habitat is the same as the tigers’
D. It is a member of the same scientific classification as the tiger.
3. What would most likely happen to a predator that wanted to eat a tiger moth?
A. The predator would be unable to catch it.
B. The predator would capture it by poisoning it.
C. The predator would be unable to find it.
D. The predator would back away from it.
4. Which of the following would a predator be most likely to attack successfully?
A. A purple and orange moth
B. a green and blur moth
C. a brown and grey moth
D. a red and yellow moth

11
VII – INFER RHETORICAL PURPOSE
You may be asked to explain why the author includes certain words, phrases, or
sentences in a passage. You must decide which of four multiple choice answers best
explains why the author choose to include bold information. Because you are asked about
the rhetorical purpose for a certain piece of information, you must look at how the bold
information fits into the overall presentation of ideas in the passage rather than only look
at the bold information itself.
The following chart outlines the key information that you should remember about
questions testing rhetorical purpose.

QUESTIONS ABOUT RHETORICAL PURPOSE


How to identify the Why does the author…?
question The author mentions X in order to…
Where to find the
The targeted information is bold in the passage.
answer
How to answer the 1. Study the bold information carefully.
question 2. Study the context around the bold information and ask
yourself how the highlighted information is related to the
context around it.
3. Draw a conclusion about the purpose of the bold
information.
4. Read the answer choices, and eliminate any definitely
wrong answers.
5. Choose the best answer from the remaining choices.
Ex:
As important as it was to US history, the dependency on cotton was not unique to the
American farmer. During the Civil War, the North blockaded the Southern ports, making
it impossible for them to ship their cotton to England. England, still needing cotton for its
mills, turned to Egypt for its cotton supply. The British and Egyptian governments invested
heavily in setting up cotton farms. However, when the American Civil War ended and
cotton production resumed, the British abandoned the Egyptian cotton fields in favor of
the much cheaper American cotton. This sent the Egyptian economy into a collapse which
resulted in the country declaring bankruptcy in 1876.
In the paragraph, the author mentions Egypt in order to
A. illustrate how English farmers avoided a dependency on cotton.
B. illustrate that dependency on cotton was a problem for many economies.
C. show how war affected cotton based economies
D. show why American farmers had to sell their cotton cheaply.

12
Practice 1
In the study of theatre, staging – the way a play actually looks onstage – is extremely
important. Staging can include scenery and props as well as costumes, lighting, and
sound. The concept of staging has changed markedly over the years.
The Elizabethan theater of Shakespeare’s day used almost no scenery or props, with one
notable exception. Unlike plays that came before, costumes of the early 1600s were often
quite elaborate. Indeed, many audience members came to the plays just to see the
costumes worn by the actors. Otherwise, an actor’s prop might be nothing more than a
sword or a crown. Chairs or tables were commonly painted on backdrops. The setting
was described through the actor’s words, not through scenery or props onstage.
This all changed in the late 1800s. Now, rather than seeing a wall painted on a backdrop,
the audience could see a real wall, made of wood and complete with a door and windows,
onstage. Lighting, which had come from natural sources (typically the sun) in Elizabethan
times, was now achieved through limelight, which created bright light by passing a flame
through a cylinder of lime. Music and sound effects also influenced staging. Music added
to the mood and meaning of a play helping the audience better understand the action
onstage.
1. Why does the author include ‘the way a play actually looks onstage’ in the passage?
A. to compare two staging method
B. to define a key term
C. to list a step in a process
D. to explain an aspect of staging
2. Why does the author mention real walls in paragraph 3?
A. to explain the effects of walls upon lighting design
B. to argue that sets in the 1800s were not artistic
C. to demonstrate how painting was used in scenery
D. to show how sets had evolved by the late 1800s.

Practice 2
One more familiar use of electrochemistry that has made its way into the mainstream is
xerography, a process for replicating documents that is dependent on photoconductive
materials. A photoconductive material is an insulator in the dark but becomes a
conductor when exposed to bright light. When a photocopy is being made, and image
of a document is projected onto the surface of a rotating drum, and bright light causes the
photoconductive material on the surface of the drum to become conductive.
As a result of the conductivity, the drum loses its charge in the lighted areas, and toner
(small grains to which dry ink adheres) attaches itself only to the darker parts of the
image. The grains are then carried to a sheet of paper and fused with heat. When a laser
printer is used, the image is projected by means of a laser beam, which creates a burgher

13
light and a greater contrast between lighter and darker areas and therefore results in a
sharper printer images.
1. Why does the author explain that ‘A photoconductive material is an insulator in the
dark but becomes a conductor when exposed to bright light’?
A. It gives an explanation of a property that is necessary for xerography.
B. It indicates that bright light is required for insulation to take place.
C. It gives one example of a successful xerographic process.
D. It explains the role of insulation in xerography.
2. The author places the phrase ‘small grains to which dry ink adheres’ in parentheses
in order to
A. provide information that contradicts the previous statement
B. provide another example of conductivity
C. provide further detail information about toner
D. provide an alternate explanation for the effectiveness of toner
VIII – INSERT SENTENCES INTO THE PASSAGE
You may be asked to determine where to insert a sentence into a passage. This type of
question asks you to decide where a sentence could be added to one of the paragraphs.
To answer this question, you should study the sentence to be inserted and then look at
the context before and after each insertion position.
How to answer the question:
1. Look at the sentence to be inserted for any key words or ideas at the beginning or
the end of the sentence.
2. Read the context before and after the insertion squares for any ideas that relate to
the sentence to be inserted.
3. Choose the insertion square that is most related to the sentence to be inserted.
Ex: For the general population, taking additional dietary supplements is for the most part
a waste of time and money. [A] Moreover, taking too many supplements can even be
harmful. [B] Eating a balanced diet alone will provide the average person with all the
essential nutrients. [C] There are some exceptions to this general rule, however. [D]
Older people with a limited exposure to sunlight may need to take vitamin D supplements,
and women of childbearing age may want to supplement their diet with folic acid to reduce
the risk of birth defects.
Where can the following sentence be added to the paragraph?
‘This is because most supplements contain many times the recommended daily
levels of vitamins and minerals’.
A. [A] C. [C]
B. [B] D. [D]

14
Practice 1
[A] One method of popping corn involved skewering an ear of corn on a stick and roasting
it until the kernels popped off the ear. [B] Corn was also popped by first cutting the kernels
off the cob, throwing them into a fire, and gathering them as they
popped out of the fire. [C] In a final method for popping corn, sand and unpopped kernels
of corn were mixed together in a cooking pot and heated until the corn popped to the
surface of the sand in the pot.[D]
Where can the following sentence be added to the paragraph?
‘Native Americans have been popping corn for at least 5000 years, using a variety
of different methods’.
A. [A] C. [C]
B. [B] D. [D]
Practice 2
[A] This traditional Native American dish was quite a novelty to newcomers to the
Americas. [B] Columbus and his sailors found natives in the West Indies wearing popcorn
necklaces, and explorer Hernando Cortes described the use of popcorn amulets in the
religious ceremonies of the Aztecs. [C] According to legendary descriptions of the
celebratory meal, Quadequina, the brother of Chief Massasoit, contributed several
deerskin bags of popcorn to the celebration.[D]
Where can the following sentence be added to the paragraph?
‘A century after these early explorers, the Pilgrims at Plymouth may have been
introduced to popcorn at the first Thanksgiving dinner.
A. [A] C. [C]
B. [B] D. [D]
IX - RECOGNIZE THE ORGANIZATION
In this type of question, you will be asked to determine how the ideas in one paragraph
(or paragraphs) related to the ideas in another paragraph (or paragraphs)
QUESTIONS ABOUT ORGANIZATION OF IDEAS
How to How is the information in the passage organized?
identify the How is the information in the second paragraph related to the
question
information in the first paragraph?
Where to find The answer to this type of question can generally be determined
the answer by looking at the first sentence of the appropriate paragraphs
How to answer 1. Read the first line of each paragraph
the question 2. Look for words that show the relationship between the
paragraph
3. Choose the answer that best expresses the relation ship

15
Ex:
If asked who invented the game of baseball, most Americans would probably reply that it
was Abner Doubleday. At the beginning of this century, there was some disagreement
over how the game of baseball had actually originated, so sporting-goods manufacturer
Spaulding inaugurated a commission to research the question. In 1908 a report was
published by the commission in which Abner Doubleday, a US Army officer from
Cooperstown, New York, was given credit for the invention of the game. The National
Baseball Hall of Fame was established in Cooperstown in honor of Doubleday.
Today, most sports historians are in agreement that Doubleday really did not have much
to do with the development of baseball. Instead, baseball seems to be a close relation to
the English game of rounders and probably has English rather than American roots.
The second paragraph
A. provides examples to support the ideas in the first paragraph
B. precedes the ideas in the first paragraph
C. outlines the effects of the ideas in the first paragraph
D. refutes the idea in the first paragraph
Also, you will sometimes be asked to determine what probably came before the reading
passage (in the preceding paragraph) or what probably comes after the reading passage
(in the following paragraph). Of course, the topic of the preceding or following paragraph
is not directly stated and you must draw a conclusion to determine what is probably in
these paragraphs.
A transition question asks you to demonstrate you understand that a good writing contains
transitions from one paragraph to the next.

TRANSITION QUESTIONS
How to identify - The paragraph preceding the passage probably …
the question - What is most likely in the paragraph following the
passage?
Where to find the The answer can be found in the first line of the passage for
answer the preceding question, and in the last line for the following
question.
How to answer 1. Read the first line for a preceding question.
the question 2. Read the last line for a following question.
3. Draw a conclusion about what comes before or after.
4. Choose the answer that is reflected in the first or last line
in the passage.

16
Ex:
Another myth of the oceans concerns Davy Jones, who in folklore is the mean-spirited
sovereign of the ocean’s depths. The name “Jones” is thought by etymologists to have
been derived from the name “Jonah”, the Hebrew prophet who spent three days in a
whale’s belly.
According to tradition, any object that goes overboard and sinks to the bottom of the
ocean is said to have gone to Davy Jones’ locker; the ocean-sized mythical receptacle
for anything that falls into the water. Needless to say, any sailor on the seas is not so
eager to take a tour of Davy Jones’ locker, although it might be a rather interesting trip
considering all the treasures located there.
The paragraph preceding this passage most probably discusses
A. The youth of Davy Jones
B. Davy Jones’ career as a sailor.
C. A different traditional story from the sea
D. Preparing to travel on the sea.
Practice 1
IQ, or Intelligence Quotient, is defined as the ratio of a person’s mental age to
chronological age, with the ratio multiplied by 100 to remove the decimal. Chronological
age is easily determined; mental age is generally measure by some kind of standard test
and is not simple to define.
In theory, a standardized IQ test is set up to measure an individual’s ability to perform
intellectual operation such as reasoning and problem solving. These intellectual
operations are considered to represent intelligence.
In practice, it has been impossible to arrive at consensus as to which types of intellectual
operations demonstrate intelligence. Furthermore, it has been impossible to devise a test
without cultural bias, which is to say that any IQ tests so far proposed have been shown
to reflect the culture of the test makers. Test takers from that culture would, it follows,
score higher on such a test than test takers from a different culture with equal intelligence.
1. What type of information is included in the first paragraph?
A. an argument B. A definition
C. An opinion D. A theory
2. How does the information in the third paragraph differ from that in the second
paragraph?
A. It presents a contrasting point of view.
B. It follows chronologically from the ideas in the second paragraph.
C. It presents real information rather than a premise.
D. It presents an example of the ideas in the second paragraph.

17
Practice 2
When a strong earthquake occurs on the ocean floor rather than on land, a tremendous
force is exerted on the seawater and one or more large destructive waves called tsunamis
can be formed. Tsunamis are commonly called tidal waves in the United States, but this
is really an inappropriate name in that the cause of the tsunami is an underground
earthquake rather than the ocean’s tides.
Far from land, a tsunami can move through the wide open vastness of the ocean at a
speed of 600 miles (900 kilometers) per hour and often travel tremendous distances
without losing height and strength. When a tsunami reaches shallow coastal water, it can
reach a height of 100 feet (30 meters) or more and can cause tremendous flooding and
damage to coastal areas.
1. The paragraph preceding the passage most probably discusses
A. Tsunamis in various parts of the world
B. The negative effects of tsunamis
C. Land-based earthquakes
D. The effects of tides on tsunamis
2. Which of the following is most likely the topic of the paragraph following the passage?
A. The causes of tsunamis
B. The destructive effects of tsunamis on the coast
C. The differences between tsunamis and tidal waves
D. The distances covered by tsunamis

X – IDENTIFY MAIN IDEA/TOPIC


You may be asked to choose a sentence that the overall topic is given. To answer this
type of question, you must be able to recognize the rhetorical pattern of the information
in the passage (i.e. compare and contrast, cause and effect, argument supported by
reasons), including the major ideas and the critical supporting information.
How to identify the questions:
- What is the best title of…?
- What would be the best/most suitable headline for the article?
- Which title best summarizes the main idea of the passage?
- What title is best for…?
- What would be the best title for…?
- What is the … most/ mainly about?

18
Where to find the answer:
The main idea or topic sentence is usually near the beginning/ the end of the passage,
pointing out the author’s main idea of the whole passage. Sometimes, there can be
exceptions, and in this case you have to skim the passage to get the general idea of the
author.
Ex:
Basketball was invented in 1891 by a physical education instructor in Springfield,
Massachusetts, by the name of James Naismith. Because of the terrible weather in
winter, his physical education students were indoors rather than outdoors. They really did
not like the idea of boring, repetitive exercises and preferred the excitement and challenge
of a game. Naismith figured out a team sport that could be played indoors on a
gymnasium floor, that involved a lot of running, that kept all team members involved, and
that did not allow the tackling and physical contact of American-style football.
What is the main idea of this passage?
A. The life of James Naismith
B. The history of sports
C. Physical education and exercise
D. The origin of basketball

Practice 1
Many people think of heat and sand when imagining a desert, but surprisingly, Antarctica
is technically the largest desert on Earth. A desert is defined as a place that gets an
extremely low amount of rainfall. A land of cold and ice, Antarctica gets less precipitation
than any other area in the world. Totaling over thirteen million square kilometers, the
Antarctic desert is almost four million square kilometers larger than the world-famous
Sahara desert.
Which title best summarizes the passage?
A. Lands of heat and sand
B. The earth’s many deserts
C. The South Pole and Antarctica
D. The vast desert of Antarctica
Practice 2
Basketball was born one day in late December of 1891. Dr. James Naismith, a Canadian
gym teacher, was trying to find a way to keep the young men in his class active during
the cold and wet winters of Massachusetts. After considering the suggestions of others
and refusing to play some of the rougher games, Naismith created his own game. After
writing down the rules, Naismith nailed a peach basket to the gym wall and told his class
they were going to play a new game.

19
The students enjoyed the game, and it became more and more popular. However, there
was a problem with using the peach basket. Every time players scored, the ball would be
stuck in the basket until someone climbed up to get it out. Over time, they learned to first
cut holes in the baskets and then poke the balls out with a long stick. Eventually, the
baskets were removed all together, replaced with metal hoops and nets. As time went on,
there were more people who were fond of playing basketball and it spread across the
country. In 1895, the first five-on-five intercollegiate basketball game was played in
Minnesota. The Basketball Association of America (BAA), which would become the
National Basketball Association (NBA), was created in 1946. Over the century, basketball
would grow and become one of the most popular American sports.
What would be the best title for the passage?
A. The history of basketball
B. A set of rules of basketball
C. The life of James Naismith
D. Advances in basketball technology

XI - DETERMINE THE TONE, PURPOSE, OR COURSE


A question about the tone is asking if the author is showing any emotion in his or her
writing. On the test, the author shows some emotion, and you must be able to recognize
that emotion to answer a question about tone correctly. If the author is funny, then the
tone might be humorous; if the author is making fun on something, the tone might be
sarcastic; if the author feels strongly that something is right or wrong, the tone might be
impassioned.
A question about the purpose is asking what the author is trying to do in the passage.
You can draw a conclusion about the author’s purpose by referring to the main idea and
the organization of details in the passage. However, the answer to a purpose question is
often considerably more general than the main idea.
A question about the courseis asking you to decide which university course might have
this passage as assigned reading. You should draw a conclusion about the course by
referring to the topic of the passage and the organization of details.
Ex:
Truman Capote’s In Cold Blood (1966) is a well-known example of the “nonfiction novel”,
a popular type of writing based upon factual events in which the author attempts to
describe the underlying forces, thoughts, and emotions that lead to actual events. In
Capote’s book, the author describes a sadistic murder of a family on a Kansas farm, often
showing the point of view of the killers. To research the book, Capote interviewed the
murderers, and he maintains that his book presents a faithful reconstruction of the
incident.

20
1. The purpose of this passage is to
A. Discuss an example of a particular literary genre.
B. Tell the story of In Cold Blood
C. Explain Truman Capote’s reasons for writing In Cold Blood
D. Describe how Truman Capote researched his nonfiction novel
2. Which of the following best describes the tone of the passage?
A. Cold B. Sadistic
C. Emotional D. Descriptive
3. This passage would probably be assigned reading in which of the following courses?
A. Criminal Law
B. American History
C. Modern American Novels
D. Literary Research
Practice 1
Up to now, confessions that have been obtained from defendants in hypnotic state have
not been admitted into evidence by court in the United States. Experts in the field of
hypnosis have found that such confessions are not completely reliable. Subjects in a
hypnosis state may confess to crimes they did not commit for one or two reasons. Either
they fantasize that they committed the crimes or they believe that others want them to
confess.
A landmark case concerning a confession obtained under hypnosis went all the way to
the US Supreme Court. In the case of Layra v. Denno, a suspect was hypnotized by a
psychiatrist for the district attorney; in a posthypnotic state the suspect signed three
separate confessions to a murder. The Supreme Court rules that the confessions were
invalid because the confessions had been the only evidence against him.
1. Which of the following best describes the author’s purpose in this passage?
A. To explain the details of a specific court case
B. To demonstrate why confessions made under hypnosis are not reliable
C. To clarify the role of Supreme Court in invalidating confessions from hypnotized
subjects
D. To explain the legal status of hypnotically induced confessions
2. The tone of this passage could best be described as
A. outraged
B. judicial
C. hypnotic
D. informative

21
3. This passage would probably be assigned reading in a course on
A. American law
B. Psychiatric healing
C. Parapsychology
D. Philosophy
Practice 2
The rate at which the deforestation of the world is proceeding is alarming. In 1950
approximately 25 percent of the Earth’s land surface had been covered with forests and
less than twenty five years later the amount of forest land was reduced to 20 percent.
This decrease from 25 to 20 percent from 1950 to 1973 represents an astounding 20
million square kilometers of forests. Predictions are that an additional 20 million square
kilometers of forest land will be lost by 2020.
The majority of deforestation is occurring in tropical forests in developing countries, fueled
by the developing countries’ need for increased agricultural land and the desire on the
part of developed countries to import wood and wood products. More than 90
percent of the plywood used in the United States, for example, is imported from
developing countries with tropical rain forests. By the mid-1980s, solutions to this
expanding problem were being sought, in the form of attempts to establish an
international regulatory organization to oversee the use of tropical forests.
1. The author’s main purpose in this passage is
A. to cite statistics about an improvement on the Earth’s land surface
B. to explain where deforestation is occurring
C. to make the reader aware of a worsening world problem
D. to blame developing countries for deforestation
2. Which of the following best describes the tone of the passage?
A. concerned C. placid
B. disinterested D. exaggerated
3. This passage would probably be assigned reading in which of the following courses?
A. Geology C. Geometry
B. Geography D. Marine Biology

22
PRACTICE TEST 1
PASSAGE 1 - Questions 1-10
When the United Nations was first formed in 1945, only 51 countries were members.
Now, however, the United Nations is truly a world organization. In fact, today almost every
nation in the world is a member of the UN. The main goal of the United Nations is to bring
different nations together to promote peace and justice in the world. The UN also works
to make the world a safe and secure place.
It is important to remember that the UN is not a ‘world government’. This means that the
UN does not make laws for different countries to follow. It also does not enforce laws
made by governments. However, the UN does hold regular votes on global policies and
issues. Also, like a government, the UN is divided into different branches, or sections.
Considering that the UN is such a large organization, it makes sense that it needs to be
separated into different pieces to be effective. There are six branches in the United
Nations. Below, the first three branches of the UN are discussed. In a later chapter, the
other three divisions will be explained.
The main branch is called the “General Assembly”. In this branch, all members of the
United Nations are represented. Each member country has one vote. These votes are
counted when the UN has meetings about world issues. For example, if there is a problem
in a certain area of the world or a particular country, the UN will vote on how to best solve
the problem. At least two-thirds of all member countries, that is 67 percent, must
agree on how to resolve the problem in order for the UN to take action. If less than
two-thirds of the voting countries agree, no immediate action is taken.
Another branch of the UN is the Security Council. [A] The main purpose of this
department is to maintain international peace and keep the world secure.  [B] In this
branch, there are only fifteen members. Five of these members are permanent. The
permanent members are China, France, Russia, the United Kingdom (Britain), and the
United States. [C] The other ten members are elected by the General Assembly for
two-year terms. [D]
The third important branch of the UN is the Economic and Social Council. This branch
works to help monitor the world economy. It also works to solve social issues around the
world. For example, issues of concern for the Economic and Social Council are violations
of human rights, the fight against international crime such as selling illegal drugs, and
destruction of the environment. There are 54 government representatives serving on this
council. These members are elected to represent certain areas of the world, so the
council has fourteen members from Africa, eleven from Asia, ten from South America,
and nineteen from Europe and North America.

1. The word ‘branches’ in the paragraph 2 is closest in meaning to


A. growing parts B. locations
C. divisions D. places

23
2. The word ‘maintain’ in paragraph 4 is closest in meaning to
A. hold on to B. preserve
C. develop D. argue
3. The pronoun ‘it’ in paragraph 5 refers to
A. the world economy
B. the general assembly of the UN
C. the monitors
D. the Economic and Social Council
4. How does the author argue that the UN is a true global organization?
A. There are 51 nations involved.
B. It allows each country to vote.
C. It includes almost all countries in the world.
D. It acts as a global government.
5. According to the passage, where does the largest percentage of representatives on
the Economic and Social Council come from?
A. South America
B. Asia
C. Europe and North America
D. Africa
6. According to the passage, how many members of the Security Council are elected on
a rotating basis?
A. 15 B. 5
C. 20 D. 10
7. Which of the sentences below best expresses the essential information in the bold
sentence in paragraph 3? Incorrect choices change the meaning in important ways or
leave out essential information.
A. The United Nations cannot take action right away because the General
Assembly always has to meet together and vote.
B. At least 66 percent of the General Assembly votes whenever the UN is asked
to take action.
C. A certain number of members must vote the same way in order for the assembly
to do something.
D. Countries that vote in the General Assembly of the UN seldom agree, so
immediate action on issues is not taken.

24
8. All of the following statements are true EXCEPT
A. The Economic and Social Council has fewer members than the Security Council.
B. There are six branches of the UN.
C. China is one of the five permanent members of the Security Council.
D. The UN tries to promote world issue and peace.
9. What does the author imply when he explains that the UN is not a world government?
A. The UN would probably work better if it did function as a government.
B. Some larger countries would like the UN to be a government.
C. Many people mistakenly think that the UN is really a type of government.
D. Countries do not like governments.
10. Where can the following sentence be added to the passage?
If the Council feels that international peace is being threatened, the fifteen
members will try to outline a way to solve the situation in a peaceful manner.
A. [A] B. [B]
C. [C] D. [D]

PASSAGE 2 - Questions 11-20


Sometimes it’s hard to figure out if you have a food allergy, since it can show up so many
different ways. Your symptoms could be caused by many other problems. You may have
rashes, hives, joint pains mimicking arthritis, headaches, irritability, or depression. The
most common food allergies are to milk, eggs, seafood, wheat, nuts, seeds, chocolate,
oranges, and tomatoes. Many of these allergies will not develop if these foods are not
fed to an infant until her or his intestines mature at around seven months. Breast milk
also tends to be protective. Migraines can be set off by foods containing tyramine,
phenathylamine, monosodium glutamate, or sodium nitrate. Common foods which
contain these are chocolate, aged cheeses, sour cream, red wine, pickled herring,
chicken livers, avocados, ripe bananas, cured meats. Some people have been successful
in treating their migraines with supplement of B-vitamins, particularly B6 and niacin.
Children who are hyperactive may benefit from eliminating food additives, especially
colorings, and foods high in salicylates from their diets. A few of these are almonds,
green peppers, peaches, tea, grapes. This is the diet made popular by Benjamin
Feingold, who has written the book ‘Why your Child is Hyperactive’. Other researchers
have had mixed results when testing whether the diet is effective.

25
11. The topic of this passage is
A. reactions to foods
B. food and nutrition
C. infants and allergies
D. a good diet
12. According to the passage, the difficulty in diagnosing allergies to foods is due to
A. the vast number of different foods we eat
B. lack of proper treatment plan
C. the similarity of symptoms of the allergy to other problems
D. the use of prepared formula to feed babies
13. The word “symptom” is closest in meaning to
A. Indications B. Diet
C. Diagnosis D. Prescriptions
14. The phrase “set off” is closest in meaning to
A. Relieved B. Identified
C. Avoided D. Triggered
15. What can be inferred about babies from this passage?
A. They can eat almost nothing.
B. They should have a carefully restricted diet as infants.
C. They gain little benefit from being breast fed.
D. They may become hyperactive if fed solid food too early.
16. The word “hyperactive” is closest in meaning to
A. overly active B. unusually low activity
C. Excited D. Inquisitive
17. The author states that the reason that infants need to avoid certain foods related to
allergies has to do with the infant’s
A. lack of teeth B. poor metabolism
C. underdeveloped intestinal tract D. inability to swallow solid foods
18. The word “these” refers to
A. food additives B. food colorings
C. innutritious foods D. foods high in salicylates

26
19. Which of the following was a suggested treatment for migraines in the passage?
A. Eating more ripe bananas
B. Avoiding all Oriental foods
C. Getting plenty of sodium nitrate
D. Using Vitamin B in addition to a good diet
20. According to the passage the Feingold diet is NOT
A. verified by researchers as being consistently effective
B. available in book form
C. beneficial for hyperactive children
D. designed to eliminate foods containing certain food additives

PASSAGE 3 – Questions 21 – 30
If we believe that clothing has to do with covering the body, and costume with the choice
of a particular form of garment for a particular use, then we can say that clothing depends
primarily on such physical conditions as climate, health, and textile manufacture, whereas
costume reflects social factors such as religious beliefs, aesthetics, personal status, and
the wish to be distinguished from or to emulate our fellows.
The ancient Greeks and the Chinese believed that we first covered our body for some
physical reason such as protecting ourselves from the elements. Ethnologists and
psychologist have invoked psychological reasons: modesty in the case of the ancients,
and taboo, magical influence and the desire to please for the moderns.
In early history, costume must have fulfilled a function beyond that of simple utility,
perhaps through some magical significance, investing primitive man with the attributes
of other creatures. Ornaments identified the wearer with animals, gods, heroes or other
men. This identification remains symbolic in more sophisticated societies. We should bear
in mind that the theater has its distant origins in sacred performances, and in all periods
children at play have worn disguises, so as to adapt gradually to adult life.
Costume helped inspire fear or impose authority. For a chieftain, costume embodied
attributes expressing his power, while a warrior’s costume enhanced his physical
superiority and suggested he was superhuman. In more recent times, professional or
administrative costume has been devised to distinguish the wearer and to express
personal or delegated authority; this purpose is seen clearly in the judge’s robes and the
police officer’s uniform. Costume denotes power, and since power is usually equated
with wealth, costume came to be an expression of social caste and material prosperity.
Military uniform denotes rank and is intended to intimidate, to protect the body and to
express membership in a group. At the bottom of the scale, there are such compulsory
costumes as the convict’s uniform. Finally, costume can possess a religious significance
that combines various elements: an actual or symbolic identification with a god, the desire
to express this in earthly life, and the desire to enhance the wearer’s position of respect.

27
21. The passage mainly discusses costume in terms of its
A. physical protection
B. religious significance
C. social function
D. beauty and attractiveness
22. What is the purpose of paragraph 1?
A. To describe the uses of costume
B. To contrast costume with clothing
C. To trace the origins of costume
D. To point out that clothing developed before costume
23. Psychological reasons for wearing garments include
A. protection from cold
B. availability of materials
C. prevention of illness
D. wishing to give pleasure
24. The word “investing” in paragraph 3 could best be replaced by
A. endowing B. creating
C. wrapping D. frightening
25. The word “Ornaments” in paragraph 3 is closest meaning to
A. garments B. representations
C. details D. decorations
26. It can be inferred from paragraph 3 that
A. the function of costume has become very sophisticated
B. children like to identify with other creatures by wearing costumes
C. primitive people wore clothes only for sacred performances
D. costume no longer fulfills a function beyond simple utility
27. Why does the author mention “the police officer’s uniform” in paragraph 4?
A. To illustrate the aesthetic function of costume
B. To identify the wearer with a hero
C. To suggest that police are superhuman
D. To show how costume signifies authority

28
28. The word “denotes” in paragraph 4 is closest meaning to
A. disguises B. describes
C. indicates D. denigrates
29. The word “scale” in paragraph 4 refers to
A. symbolic identification
B. military rank
C. social position
D. the balance
30. Which of the following would most likely NOT be reflected in a person’s costume, as
it is defined in the passage?
A. Having a heart condition
B. Playing in a baseball game
C. Working in a hospital
D. Participating in a religious ceremony

PASSAGE 4 – Questions 31 – 40

Harvard University, today recognized as part of the top echelon of the world’s universities,
came from very inauspicious and humble beginnings.
This oldest of American universities was founded in 1636, just sixteen years after the
Pilgrims landed at Plymouth. Included in the Puritan emigrants to the Massachusetts
colony during this period were more than 100 graduates of England’s prestigious Oxford
and Cambridge universities, and these universities graduates in the New World were
determined that their sons would have the same educational opportunities that they
themselves had had. Because of this support in the colony for an institution of higher
learning, the General Court of Massachusetts appropriated 400 pounds for a college in
October of 1936 and early the following year decided on a parcel of land for the school;
this land was in an area called Newtown, which was later renamed Cambridge after its
English cousin and is the site of present-day university.
When a young minister named John Harvard, who came from the neighboring town of
Charlestown, died from tuberculosis in 1638, he willed half of his estate of 1,700 pounds
to the fledgling college. In spite of the fact that only half of the bequest was actually paid,
the General Court named the college after the minister in appreciation for what he had
done. The amount of the bequest may not have been large, particularly by today’s
standards, but it was more than the General Court had found it necessary to appropriate
in order to open the college.
Henry Dunster was appointed the first president of Harvard in 1640, and it should be
noted that in addition to serving as president, he was also the entire faculty, with an
entering freshman class of four students. Although the staff did expand somewhat, for

29
the first century of its existence the entire teaching staff consisted of the president and
three or four tutors.
The main idea of this passage is that
31.
A. Harvard is one of the world’s most prestigious universities
B. what is today a great university started out small
C. John Harvard was key to the development of a great university
D. Harvard University developed under the auspices of the General Court of
Massachusetts
32. The passage indicates that Harvard is
A. one of the oldest universities in the world
B. the oldest university in the world
C. one of the oldest universities in America
D. the oldest university in America
33. It can be inferred from the passage that the Puritans who traveled to the
Massachusetts colony were
A. rather well educated
B. rather rich
C. rather supportive of the English government
D. rather undemocratic
34. The pronoun “they” in paragraph 2 refers to
A. Oxford and Cambridge universities
B. universities graduates
C. sons
D. educational opportunities
35. The word “pounds” in paragraph 2 are probably
A. types of books
B. college students
C. units of money
D. school campuses
36. The “English cousin” in paragraph 2refers to a
A. city B. relative
C. person D. court

30
37. Which of the following is NOT mentioned about John Harvard?
A. What he died of
B. Where he came from
C. Where he was buried
D. How much he bequeathed to Harvard
38. The word “fledgling” in paragraph 3 could best be replaced by
A. Newborn
B. Flying
C. Winged
D. Established
39. The passage implies that
A. Henry Dunster was an ineffective president
B. Someone else really served as president of Harvard before Henry Dunster
C. Henry Dunster spent much of his time as president managing the Harvard
faculty
D. The position of president of Harvard was not merely an administrative
position in the early years
40. The word “somewhat” in paragraph 4 could best be replaced by
A. back and forth
B. to and fro
C. side by side
D. more or less

PRACTICE TEST 2
PASSAGE 1 – Questions 1 – 10
Esperanto is what is called a planned, or artificial, language. It was created more than a
century ago by Polish eye doctor Ludwik Lazar Zamenhof. Zamenhof believed that a
common language would help to alleviate some of the misunderstanding among cultures.
In Zamenhof’s first attempt at a universal language, he tried to create a language that
was as uncomplicated as possible. This first language included words such as ab, ac, ba,
eb, be, and ce. This did not result in a workable language in that these monosyllabic
words, though short, were not easy to understand or to retain.
Next, Zamenhof tried a different way of constructing a simplified language. He made the
words in his language sound like words that people already knew, but he simplified the
grammar tremendously. One example of how he simplified the language can be seen in

31
the suffixes: all nouns in this language end in o,as in the noun amiko, which means
“friend,” and all adjectives end in –a, as in the adjective bela, which means “pretty”.
Another example of the simplified language can be seen in the prefix mal-, which makes
a word opposite in meaning; the word malamiko therefore means “enemy”, and the word
malbela therefore means “ugly” in Zamenhof’s language.
In 1887, Zamenhof wrote a description of this language and published it. He used a pen
name, Dr. Esperanto, when signing the book. He selected the name Esperanto because
this word means “a person who hopes” in his language. Esperanto clubs began popping
up throughout Europe, and by 1905, Esperanto had spread from Europe to America and
Asia.
In 1905, the First World Congress of Esperanto took place in France, with approximately
700 attendees from 20 different countries. Congresses were held annually for nine years,
and 4,000 attendees were registered for the Tenth World Esperanto Congress scheduled
for 1914, when World War I erupted and forced its cancellation.
Esperanto has had its ups and downs in the period since World War I. Today, years after
it was introduced, it is estimated that perhaps a quarter of a million people are fluent in it.
This may seem like a large number, but it is really quite small when compared with the
billion English speakers and billion Mandarin Chinese speakers in today’s world. Current
advocates would like to see its use grow considerably and are taking steps to try to make
this happen.
1. The topic of this passage is
A. a language developed in the last few years
B. one man’s efforts to create a universal language
C. how language can be improved
D. using language to communicate internationally
2. According to the passage, Zamenhof wanted to create a universal language
A. to resolve cultural differences
B. to provide a more complex language
C. to build a name for himself
D. to create one world culture
3. It can be inferred from the passage that the Esperanto word malespera
means
A. hopelessness B. hope
C. hopeless D. hopeful
4. The expression “popping up” in paragraph 4 could best be replaced by
A. leaping B. shouting
C. hiding D. opening

32
5. It can be inferred from the passage that the Third World Congress of
Esperanto took place
A. in 1905 B. in 1907
C. in 1909 D. in 1913
6. According to the passage, what happened to Tenth World Esperanto
Congress?
A. It had 4,000 attendees.
B. It was scheduled for 1915.
C. It had attendees from 20 countries.
D. It never took place.
7. The expression “ups and downs” in paragraph 6 is closest in meaning to
A. tops and bottoms
B. floors and ceilings
C. takeoffs and landings
D. highs and lows
8. Which paragraph describes the predecessor to Esperanto?
A. The first paragraph
B. The second paragraph
C. The third paragraph
D. The last paragraph
9. This passage would most likely be assigned reading in a course on
A. European history
B. English grammar
C. world government
D. applied linguistics
10. The paragraph following the passage most likely discusses
A. how current supporters of Esperanto are encouraging its growth
B. another of Zamenhof’s accomplishments
C. the disadvantages of using an artificial language
D. attempts to reconvene the World Congress of Esperanto in the 1920s

33
PASSAGE 2 – Questions 11 -20

Crows are probably the most frequently met and easily identifiable members of the native
fauna of the United States. The great number of tales, legends, and myths about these
birds indicates that people have been exceptionally interested in them for a long time. On
the other hand, when it comes to substantive – particularly behavioral – information, crow
are less well-known than many comparably common species and, for that matter, not a
few quite uncommon ones: the endangered California condor, to cite one obvious
example. There are practical reasons for this.
Crows are notoriously poor and aggravating subjects for field research. Keen observers,
and quick learners, they are astute about the intentions of other creatures, including
researchers, and adapt at avoiding them. Because they are so numerous, active, and
monochromatic, it is difficult to distinguish one crow to another. Bands, radio transmitters,
and other identifying devices can be attached to them, but this course requires catching
live crows, who are among the wariest and most untrappable of birds.
Technical difficulties aside, crow research is daunting because the ways of these birds
are so complex and various. As prominent generalists, members of this species
ingeniously exploit a great range of habitats and resources, and they can quickly adjust
to changes in their circumstances. Being so educable, individual birds have markedly
different interests and inclinations, strategies and scrams. For example, one pet crow
learned how to let a dog out of its kennel by pulling the pin on the door. When the dog
escaped, the bird went into the kennel and ate its food.
11. What is the main idea of the passage?
A. The ways in which crows differ from other common birds
B. The myths and legends about crows
C. The characteristics that make crows difficult to study
D. The existing methods for investigating crow behavior
12. According to the first paragraph, what evidence is there that crows have
interested people for a long time?
A. The large number of stories about crows
B. The frequency with which crows are sighted
C. The amount of research that has been conducted on crows
D. The ease with which crows are identified
13. The word “comparably” in paragraph 1 is closest in meaning to
A. interestingly B. similarly
C. otherwise D. sometimes
14. In paragraph 1, the author mentions “the endangered California condor” as an
example of a species that is

34
A. smaller than a crow B. easily identifiable
C. featured in legends D. very rare
15. The word “them” in paragraph 2 refers to
A. crows B. subjects
C. intentions D. researchers
16. According to the second paragraph, crows are poor subjects for field research
for all of the following reasons EXCEPT
A. They can successfully avoid observers.
B. They are hard to distinguish from one another.
C. They can be quite aggressive.
D. They are difficult to catch.
17. According to the third paragraph, which of the following is true about crows?
A. They seldom live in any one place for very long.
B. They thrive in a wide variety of environments.
C. They have marked preferences for certain kinds of foods.
D. They use up the resources in one area before moving to another.
18. In paragraph 3, the word “inclination” is closest in meaning to
A. Tricks
B. opportunities
C. preferences
D. experiences
19. In paragraph 3, the author mentions a pet crow to illustrate which of the
following?
A. The clever ways that crows solve the problems
B. The differences between pet crows and wild crows
C. The ease with crows can be tamed
D. The affection that crows show to other creatures
20. Which of the following statements is supported by the passage?
A. Crows have relatively long lives.
B. Crows have keen vision.
C. Crows are usually solitary.
D. Crows are very intelligent.

35
PASSAGE 3– Questions 21 -30

Before the mid-nineteenth century, people in the United States ate most foods only in
season. Drying, smoking, and salting could preserve meat for a short time, but the
availability of fresh meat, like that of fresh milk, was very limited: there was no way to
prevent spoilage. But in 1810 a French inventor named Nicolas Appert developed the
cooking-and-sealing process of canning. And in the 1850s, an
American named Gail Borden developed a means of condensing and preserving milk.
Canned goods and condensed milk became more common during the 1860’s, but
supplies remained low because cans had to be made by hand. By 1880, however,
inventors had fashioned stamping and soldering machines that mass-produced cans
from tinplate. Suddenly all kinds of food could be preserved and bought at all times of
the year.
Other trends and inventions had also helped make it possible for Americans to vary their
daily diets. Growing urban populations created demand that encouraged fruit and
vegetable farmers to raise more produce. Railroad refrigerator cars enabled growers
and meat packers to ship perishables great distances and to preserve them for longer
periods. Thus, by the 1890’s, northern city dwellers could enjoy southern and western
strawberries, grapes, and tomatoes, previously available for a month at most, for up to
six months of the year. In addition, increased use of iceboxes enables families to store
perishables. An easy means of producing ice commercially had been invented in the
1870’s, and by 1900 the nation had more than two thousand commercial ice plants,
1870’s, most of which made home deliveries. The icebox became a fixture in most
homes and remained so until the mechanized refrigerator replaced it in the 1920’s and
1930’s.
Almost everyone now had a more diversified diet. Some people continued to eat mainly
foods that were heavy in starches or carbohydrates, and not everyone could afford
meat. Nevertheless, many families could take advantage of previously unavailable
fruits, vegetables and dairy products to achieve more varied fare.
21. What does the passage mainly discuss?
A. Causes of food spoilage
B. Commercial production of ice
C. Population movements in the nineteenth century
D. Inventions that led to changes in the American diet
22. The phrase ‘in season’ in paragraph 1 refers to
A. A kind of weather
B. A particular time of year
C. An official schedule

36
D. A method of flavoring food
23. The word ‘prevent’ in paragraph 1 is closet in meaning to
A. Avoid B. Estimate
C. Correct D. Confine
24. During the 1860’s, canned food products were
A. Unavailable in rural areas
B. Shipped in refrigerator cars
C. Available in limited quantities
D. A staple part of the American diet
25. It can be inferred that railroad refrigerator cars came into use
A. Before 1860 B. Before 1890
C. After 1900 D. After 1920
26. The word ‘them’ in paragraph 2 refers to
A. Refrigerator cars B. Growers
C. Distances D. Perishables
27. The word ‘fixture’ in paragraph 2 is closet in meaning to
A. Luxury item B. Commonplace object
C. Substance D. Mechanical device
28. The author implies that in the 1920’s and 1930’s home deliveries of ice
A. Decreased in number
B. Were on an irregular schedule
C. Increased in cost
D. Occurred only in the summer
29. Which of the following types of food preservation was NOT mentioned in the
passage?
A. Drying B. Canning
C. Cold storage D. Chemical additives
30. Which of the following statements is supported by the passage?
A. Tin cans and iceboxes helped to make many foods more widely available.
B. Commercial ice factories were developed by railroad owners.
C. Most farmers in the United States raised only fruits and vegetables.
D. People who lived in cities demanded home delivery of foods

37
PASSAGE 4 – Questions 31 – 40
Sequoyah was a young Cherokee Indian, son of a white trader and an Indian squaw. At
an early age, he became fascinated by "the talking leaf," an expression that he used to
describe the white man's written records. Although many believed this "talking leaf' to be
a gift from the Great Spirit, Sequoyah refused to accept that theory. Like other Indians
of the period, he was illiterate, but his determination to remedy the situation led to the
invention of a unique eighty-six-character alphabet based on the sound patterns that he
heard.
His family and friends thought him mad, but while recuperating from a hunting accident,
he diligently and independently set out to create a form of communication for his own
people as well as for other Indians. In 1821, after twelve years of work, he had
successfully developed a written language that would enable thousands of Indians to
read and write.
Sequoyah's desire to preserve words and events for later generations has caused him
to be remembered among the important inventors. The giant redwood trees of California,
called "sequoias" in his honor, will further imprint his name in history.
31. What is the most important reason that Sequoyah will be remembered?
A. California redwoods were named in his honor.
B. He was illiterate.
C. He created a unique alphabet.
D. He recovered from his madness and helped humankind.
32. The word "squaw" in paragraph 1 is closest in meaning to
A. woman B. teacher C. cook D. trader
33. How did Sequoyah's family react to his idea of developing his own "talking leaf'?
A. They arranged for his hunting accident.
B. They thought he was crazy.
C. They decided to help him.
D. They asked him to teach them to read and write.
34. What prompted Sequoyah to develop his alphabet?
A. People were writing things about him that he couldn't read.
B. He wanted to become famous.
C. After his hunting accident, he needed something to keep him busy.
D. He wanted the history of his people preserved for future generations.
35. In paragraph 1, the word "illiterate" means most nearly the same as
A. fierce
B. poor

38
C. abandoned
D. unable to read or write
36. It is implied that Sequoyah called the written records "the talking leaf” because
A. they played music
B. when he observed white people reading, they seemed to understand what was
written
C. he was going mad, and he thought the leaves were talking to him
D. it was the only way that the Great Spirit had of communicating with them
37. Sequoyah could best be described as
A. determined B. mad C. backwards D. meek
38. What is the best title for the passage?
A. Sequoyah 's Determination to Preserve the Cherokee Language
B. The Origin of the Cherokee Language
C. Sequoyah's Madness Leads to a New Language
D. The Origin of the "Sequoia" Trees in California
39. In paragraph 1, "fascinated" is closest in meaning to
A. absorbed B. exasperated
C. confused D. imaginative
40. All of the following are true EXCEPT
A. Sequoyah developed a form of writing with the help of the Cherokee tribe
B. Sequoyah was a very observant young man
C. Sequoyah spent twelve years developing his alphabet
D. Sequoyah was honored by having some trees named after him

39
PRACTICE TEST 3
PASSAGE 1 – Questions 1 – 10
Niagara Falls, one of the most famous North American natural wonders, has long
been a popular tourist destination. Tourists today flock to see the two falls that
actually constitute Niagara Falls: the 173-foot-high Horseshoe Falls on the
Canadian side of the Niagara River in the Canadian province of Ontario and the
182-foot-high American Falls on the U.S. side of the river in the state of New York.
Approximately 85 percent of the water that goes over the falls actually goes over
Horseshoe Falls, with the rest going over American Falls.
Most visitors come between April and October, and it is quite a popular activity to
take a steamer out onto the river and right up to the base of the falls for a close-
up view. It is also possible to get a spectacular view of the falls from the strategic
locations along the Niagara River, such as Prospect Point or Table Rock, or from
one of the four observation towers which have heights up to 500 feet.
Tourists have been visiting Niagara Falls in large numbers since the 1980s; annual
visitation now averages above 10 million visitors per year. Because of concern that
all these tourist would inadvertently destroy the natural beauty of this scenic wonder,
the state of New York in 1885 created Niagara Falls Park in order to protect the land
surrounding American Falls. A year later Canada created Queen Victoria Park on
the Canadian side of the Niagara, around Horseshoe Falls. With the area
surrounding the falls under the jurisdiction of government agencies, appropriate
steps could be taken to preserve the pristine beauty of the area.
1. What is the major point that the author is making in this passage?
A. Niagara Falls can be viewed from either the American side or the Canadian
side.
B. A trip to the United States isn’t complete without a visit to Niagara Falls.
C. Niagara Falls has had an interesting history.
D. It has been necessary to protect Niagara Falls from the many tourists who go there.

40
2. The word “flock” in paragraph 1 could best be replaced by
A. come by plane
B. come in large numbers
C. come out of boredom
D. come without knowing what they will see
3. According to the passage, which of the following best describes Niagara Falls?
A. Niagara Falls consists of two rivers, one Canadian and the other American.
B. American Falls is considerably higher than Horseshoe Falls.
C. The Niagara River has two falls, one in Canada and one in the United States.
D. Although the Niagara River flows through the United States and Canada, the falls
are only in the United States.
4. A “steamer” in paragraph 2 is probably
A. a bus
B. a boat
C. a walkway
D. a park
5. The expression “right up” in paragraph 2 could best be replaced by
A. turn to the right
B. follow correct procedures
C. travel upstream
D. all the way up
6. The passage implies that tourists prefer to
A. visit Niagara Falls during warmer weather
B. see the falls from a great distance
C. take a ride over the falls
D. come to Niagara Falls for a winter vacation.
7. According to the passage, why was Niagara Park created?
A. To encourage tourists to visit Niagara Falls.
B. To show off the natural beauty of Niagara Falls.
C. To protect the area around Niagara Falls.
D. To force Canada to open Queen Victoria Park.

41
8. The word “jurisdiction” in paragraph 3 is closest in meaning to
A. view
B. assistance
C. taxation
D. control
9. The word “pristine” in paragraph 3 is closest in meaning to
A. pure and natural
B. highly developed
C. well-regulated
D. overused
10. The paragraph following the passage most probably discusses
A. additional ways to observe the falls
B. steps taken by government agencies to protect the falls
C. a detailed description of the division of the falls between the United States and
Canada
D. further problems that are destroying the area around the falls.

PASSAGE 2 – Questions 11 – 18
What is commonly called pepper in reality comes from two very different families of
plants. Black and white pepper both come from the fruit of the Piper nigrum, a vine
with fruits called peppercorns. The peppercorns turn from green to red as they ripen
and finally blacken as they dry out. The dried-out peppercorns are ground to obtain
black pepper. White pepper, which has a more subtle favor than black pepper,
comes from the same peppercorns as black pepper; to obtain white pepper, the
outer hull of the peppercorn, the pericarp, is removed before the peppercorn is
ground.
Red and green peppers, on the other hand, come from a completely different family
from black and white pepper. Red and green peppers are from the genus Capsicum.
Plants of this type generally have tiny white flowers and fruit which can be any one
of a number of colors, shapes, and sizes. These peppers range in flavor from very
mild and sweet to the most incredibly burning taste imaginable. Bell peppers are the
mildest, while habaneros are the most burning.
Christopher Columbus is responsible for the present-day confusion over what a
pepper is. The Piper nigrum variety of pepper was highly valued for centuries, and
high demand for pepper by Europeans was a major cause of the fifteenth-century
push to locate ocean routes to the spice-growing regions of Asia. When Columbus
arrived in the New World in 1492, he was particularly interested in finding black
pepper because of the high price that it would command in Europe. Columbus came

42
across plants from the Capsicum family in use among the people of the New World,
and he incorrectly identified them as relatives of black pepper. Columbus introduced
the spicy Capsicum chili peppers to Europeans on his return from the 1492 voyage,
and traders later spread them to Asia and Africa. These Capsicum peppers have
continued to be called peppers in spite of the fact that they are not related to the
black and white pepper of the Piper nigrum family.
11. The purpose of this passage is to
A. explain why there is confusion today over peppers
B. provide the scientific classification of various types of peppers
C. demonstrate that it was Columbus who brought peppers to Europe
D. classify the variety of sizes, shapes, and colors of peppers
12. The word "turn" in paragraph 1 could best be replaced by
A. revert B. exchange
C. veer D. change
13. According to the passage, both black and white peppers
A. come from different plants
B. change colors after they are ground
C. are ground from dried out peppercorns
D. have the same flavor
14. What part of the Piper nigrum is the pericarp?
A. the seed inside the fruit
B. the outer covering of the fruit
C. the pulp inside the vine
D. the outer covering of the vine
15. What usually does NOT vary in a Capsicum plant?
A. The color of the flower
B. The size of the fruit
C. The shape of the fruit
D. The color of the fruit
16. The word "push" in paragraph 3 could best be replaced by
A. shove
B. strength
C. drive
D. hit

43
17. The pronoun “them” in paragraph 3 refers to
A. plants
B. people
C. relatives
D. Europeans
18. It can be inferred from the passage that chili peppers originally came from
A. Europe
B. Asia
C. America
D. Africa

PASSAGE 2 – Questions 19 – 30
Just two months after the flight of Apollo 10, the Apollo 11 astronauts made their
historic landing on the surface of the Moon. This momentous trip for humanity also
provided scientists with an abundance of material for study; from rock and soil
samples brought back from the Moon, scientists have been able to determine much
about the composition of the Moon as well as to draw inferences about the
development of the Moon from its composition.
The Moon soil that came back on Apollo 11 contains small bits of rock and glass
which were probably ground from larger rocks when meteors impacted with the
surface of the Moon. The bits of glass are spherical in shape and constitute
approximately half of the Moon soil. Scientists found no trace of animal or plant life
in this soil.
In addition to the Moon soil, astronauts gathered two basic types of rocks from the
surface of the Moon: basalt and breccia. Basalt is cooled and hardened volcanic
lava common to the Earth. Since basalt is formed under extremely high
temperatures, the presence of this type of rock is an indication that the temperature
of the Moon was once extremely hot. Breccia, the other kind of rock, brought back
by the astronauts, was formed during the impact of falling objects on the surface of
the Moon. This second type of rock consists of small pieces of rock compressed
together by the force of impact. Gases such as hydrogen and helium were found in
some of the rocks, and scientists believe that these gases were carried to the Moon
by the solar wind, the streams of gases that are constantly emitted by the Sun.
19. The paragraph preceding the passage most likely discusses
A. astronaut training
B. the inception of the Apollo space program
C. a different space trip
D. previous Moon landings

44
20. What is the subject of this passage?
A. The Apollo astronauts
B. Soil on the Moon
C. What the Moon is made of
D. Basalt and breccia
21. An “abundance” in paragraph 1 is
A. a disorderly pile
B. a wealthy bunch
C. an insignificant proportion
D. a large amount
22. According to the passage, what does Moon soil consist of?
A. Hydrogen and helium.
B. Large chunks of volcanic lava
C. Tiny pieces of stones and glass
D. Streams of gases
23. The word “spherical” in paragraph 2 is closest in meaning to
A.Earthen
B. Circular
C.Angular
D. Amorphous
24. Which of the following was NOT brought back to the Earth by the astronauts?
A. basalt
B. soil
C. breccia
D. plant life
25. An “indication” in paragraph 3 is
A.an exhibition
B. a clue
C. a denial
D. a dictate

45
26. According to the passage, breccia was formed
A. when objects struck the Moon
B. from volcanic lava
C. when streams of gases hit the surface of the Moon
D. from the interaction of helium and hydrogen
27. It is implied in the passage that scientists believe that the gases found in the Moon rocks
A. were not originally from the Moon
B. were created inside the rocks
C. traveled from the Moon to the Sun
D. caused the Moon's temperature to rise
28. The word “emitted” in paragraph 3 is closest in meaning to
A. set off
B. vaporized
C. sent out
D. separated
29. The author's purpose in this passage is to
A. describe some rock and soil samples
B. explain some of the things learned from space flights
C. propose a new theory about the creation of the Moon
D. demonstrate the difference between basalt and breccia
30. It can be inferred from the passage that
A. the only items of importance that astronauts brought back from the Moon were rock
and soil samples
B. scientists learned relatively little from the Moon rock and soil samples
C. scientists do not believe that it is necessary to return to the Moon
D. rock and soil samples were only some of a myriad of significant items from the Moon

46
PASSAGE 4 – Questions 31 – 40

During the heyday of the railroads, when America's rail system provided the bulk of
the country's passenger and freight transportation, various types of railroad cars
were in service to accomplish the varied tasks handled by the railroads. One type of
car that was not available for public use prior to the Civil War; however, was a
sleeping car; ideas for sleeping cars abounded at the time, but these ideas were
unworkable. It unfortunately took the death of a president to make the sleeping car
a viable reality.
Cabinet-maker George M. Pullman had recognized the demand for sleeping cars
and had worked on developing experimental models of sleeping cars in the decade
leading up to the Civil War. However, in spite of the fact that he had made successful
test runs on the Chicago and Alton Railroads with his models, he was unable to sell
his idea because his models were too wide and too high for existing train stations
and bridges. In 1863, after spending time working as a storekeeper in a Colorado
mining town, he invested his savings of twenty thousand dollars, a huge fortune at
that time and all the money that he had in the world, in a luxurious sleeping car that
he named the Pioneer. Pullman and friend Ben Field built the Pioneer on the site of
the present-day Chicago Union Station. For two years, however, the Pioneer sat on
a railroad siding, useless because it could not fit through train stations and over
bridges.
Following President Lincoln's assassination in 1865, the state of Illinois, Lincoln's
birthplace, wanted to transport the presidential casket in the finest fashion possible.
The Pullman Pioneer was the most elegant car around; in order to make the Pullman
part of the presidential funeral train in its run from Springfield to Chicago, the state
cut down station platforms and raised bridges in order to accommodate the luxurious
railway car. The Pullman car greatly impressed the funeral party, which included
Lincoln's successor as president, General Ulysses S. Grant, and Grant later
requested the Pioneer for a trip from Detroit to Chicago. To satisfy Grant's request
for the Pioneer, the Michigan Central Railroad made improvements on its line to
accommodate the wide car, and soon other railroads followed. George Pullman
founded the Pullman Palace Car Company in partnership with financier Andrew
Carnegie and eventually became a millionaire.

31. Which of the following best states the main idea of the passage?
A. America’s railroads used to provide much of the country’s transportation.
B. President Lincoln’s assassination in 1965 shocked the nation.
C. George Pullman was the only one to come up with the idea for a sleeping car.
D. Pullman’s idea for a sleeping car became workable after Lincoln’ death.

47
32. A “heyday” in paragraph 1 is most probably
A. a time for harvest
B. a period with low prices
C. a period of great success
D. a type of railroad schedule
33. It can be inferred from the passage that before the Civil War, sleeping cars
A. were used abundantly
B. were thought to be a good idea
C. were only used privately
D. were used by presidents
34. The word “test” in paragraph 2 could best be replaced by
A. Exam
B. Trial
C. Inspection
D. Scientific
35. What was the initial problem that made Pullman’s cars unusable?
A. They were too large.
B. They were too expensive.
C. They were too slow.
D. They were too unusual.
36. What is stated in the passage about George Pullman?
A. He once had a job in a store.
B. He always lived in Chicago.
C. He worked in a mine
D. He saved money for his project.
37. The word “site” in paragraph 2 is closest in meaning to
A. Factory
B. View
C. Location
D. Foundation

48
38. Why did the state of Illinois want to use the Pullman in Lincoln’s funeral train?
A. It was superior to other cars.
B. It was the only railroad car that could make it from Springfield to Chicago.
C. Ulysses S. Grant requested it.
D. The Pullman Palace Car Company was a major Illinois business.

39. It can be inferred from the passage that the Michigan Central Railroad
A. was owned by George Pullman
B. controlled the railroad tracks between Detroit and Chicago
C. was the only railroad company to accommodate wide cars
D. was the sole manufacturer of the Pioneer
40. This passage would most likely be assigned in which of the following courses?
A. Engineering
B. Political science
C. Finance
D. History

49
PRACTICE TEST 4

PASSAGE 1 – Questions 1 – 10

The final battle of the War of 1812 was the Battle of New Orleans. This battle gave
a clear demonstration of the need for effective communication during wartime; it is
also showed the disastrous results that can come to pass when communication is
inadequate.
The War of 1812 was fought between Great Britain and the very young country of
the United States only a very few years after the United States had won its
independence from Britain. The United States had declared war against Britain in
June of 1812, mostly because of interference with U.S. shipping by the British and
because of the shanghaiing of U.S. sailors for enforced service on British vessels.
The war lasted for a little more than two years, when a peace treaty was signed at
Ghent, in Belgium, on the 24th of December, 1814.
[A] Unfortunately, the news that the Treaty of Ghent had been signed and that the
war was officially over was not communicated in a timely manner over the wide
distance to where the war was being contested. [B] Negotiations for the treaty and
the actual signing of the treaty took place in Europe, and news of the treaty had to
be carried across the Atlantic to the war front by ship. [C]
Early in January of 1815, some two weeks after the peace treaty had been signed,
British troops in the southern part of the United States were unaware that the war
had officially ended. Over 5,000 British troops attacked U.S. troops. During the
ensuing battle, known as the Battle of New Orleans, the British suffered a huge
number of casualties, around 2.000 and Americans lost 71, all in a battle fought only
because news of the peace treaty that had already been signed in Ghent had not
yet reached the battlefield. [D]
1. The main idea of this passage is that
A. the War of Independence was unnecessary
B. the War of 1812 was unnecessary
C. the Treaty of Ghent was unnecessary
D. the Battle of New Orleans was unnecessary
2. The pronoun “it” in paragraph 1 refers to
A. battle
B. demonstration
C. communication
D. wartime

50
3. The expression “come to pass” in paragraph 1could best be replaced by
A. happen
B. overthrow
C. self-destruct
D. circumvent
4. According to the passage, when did the United States win its independence from
Britain?
A. Shortly before the War of 1812
B. During the War of 1812

C. Just after the War of 1812


D. Long after the War of 1812
5. According to the passage, some U.S. sailors were
A. taken forcibly to Shanghai
B. made to go to Ghent
C. forced to work on British ships
D. responsible for causing the War of 1812
6. It is NOT stated in the passage that Ghent was
A. where negotiations took place
B. the site of the final battle
C. where the treaty was signed
D. far from the battlefield
7. The word “contested” in paragraph 3 is closest in meaning to
A. played
B. fought
C. discussed
D. examined
8. It can be determined from the passage that, of the following dates, the Battle of New
Orleans was most probably fought
A. on December 10, 1814
B. on December 24, 1814
C. on January 1, 1815
D. on January 8, 1815

51
9. Where can the following sentence be inserted in the passage?
`‘A totally unnecessary loss of life was incurred as a result of the amount of time
that it took to inform the combatants of the treaty.’
A. [A] B.[B]
C. [C] D.[D]
10. Which paragraph describes the battle that took place after the signing of the treaty?
A. The first paragraph
B. The second paragraph
C. The third paragraph
D. The last paragraph

PASSAGE 2 – Questions 11 – 20
Mount Rushmore is a well-known monument in the Black Hills of South Dakota that
features the countenances of four United States presidents: Washington, Jefferson,
Roosevelt, and Lincoln. What is not so well known is that the process of creating this
national treasure was not exactly an uneventful one.
Mount Rushmore was the project of the visionary sculptor John Gutzen de la Mothe
Borglum, who was born in Idaho but studied sculpture in Paris in his youth and
befriended the famous French sculptor Auguste Rodin. In 1927 Borglum was
granted a commission by the federal government to create the sculpture on Mount
Rushmore. Though he was nearly sixty years old when he started, he was
undaunted by the enormity of the project and obstacles that it engendered. He
optimistically asserted that the project would be completed within five years, not
caring to recognize the potential problems that such a massive project would involve,
the problems of dealing with financing, with government bureaucracy, and with
Mother Nature herself. An example of what Mother Nature had to throw at the project
was the fissure—or large crack—that developed in the granite where Jefferson was
being carved. Jefferson had to be moved to the other side of Washington, next to
Roosevelt because of the break in the stone. The work that had been started on the
first Jefferson had to be dynamited away.
Mount Rushmore was not completed within the five years predicted by Borglum and
was in fact not actually completed within Borglum’s lifetime, although it was almost
finished. Borglum died on March 6, 1941, at the age of seventy-four, after fourteen
years of work on the presidents. His son, Lincoln Borglum, who had worked with his
father throughout the project, completed the monument within eight months of his
father’s death.

52
11. Which of the following best expresses the main idea of the passage?
A. Mount Rushmore was a huge project filled with numerous obstacles.
B. Mount Rushmore is a famous American monument.
C. Mount Rushmore has sculptures of four United States presidents on it.
D. John Gutzen de la Mothe Borglum created Mount Rushmore.
12. Which of the following best describes the relationship between Borglum and Rodin in
Borglum’s early years?
A. Borglum studied about Rodin in Paris.
B. Borglum was far more famous than Rodin as a sculptor.
C. Borglum and Rodin were born and raised in the same place.
D. Borglum and Rodin were friends.
13. The word “nearly” in paragraph 2 could best be replaced by
A. Over
B. Closely
C. Almost
D. Barely
14. Which of the following is NOT true about Borglum?
A. He began Mount Rushmore around the age of sixty.
B. He predicted that Mount Rushmore would be finished around 1932.
C. Mount Rushmore was finished when Borglum predicted it would be.
D. Borglum worked on Mount Rushmore for more than a decade.
15. It can be inferred from the passage that Borglum was someone who
A. expected the best to happen
B. set realistic goals
C. never tried anything too challenging
D. was always afraid that bad things were going to happen
16. A “fissure” in paragraph 2 is
A. a discoloration
B. a break
C. an unevenness
D. a softness

53
17. Why does the author mention the fact that the carving of Thomas Jefferson was
moved?
A. It shows what a perfectionist Borglum was.
B. It demonstrates Borglum’s artistic style.
C. It gives insight into Jefferson’s character.
D. It is an example of a problem caused by nature.
18. The pronoun “it” in paragraph 3 refers to
A. The first Jefferson
B. Mount Rushmore
C. Borglum’s lifetime
D. Fourteen years of work
19. Which of the following is closest in meaning to the expression “within eight months
of his father’s death” in paragraph 3?
A. More than eight months before his father’s death
B. Less than eight months before his father’s death
C. Less than eight months after his father’s death
D. More than eight months after his father’s death
20. This passage would most likely be assigned reading in a course on
A. art history B. geography
C. management D. government

PASSAGE 3 – Questions 21 – 30
Are
' organically grown foods the best food choices? The advantages claimed for
such foods over conventionally grown and marketed food products are now being
debated. Advocates of organic foods – a term whose meaning varies greatly –
frequently proclaim that such products are safer and more nutritious than others.
The growing interest of consumers in the safety and more nutritional quality of the
typical North American diet is a welcome development. However, much of this
interest has been sparked by sweeping claims that the food supply is unsafe or in
adequate in meeting nutritional needs. Although most of these claims are not
supported by scientific evidence, the preponderance of written material advancing
such claims makes it difficult for the general public to separate fact from fiction. As
a result, claims that eating a diet consisting entirely of organically grown foods
prevents or cures disease or provides other benefits to health have become widely
publicized and form the basis for folklore.
Almost daily the public is besieged by claims for "no-aging" diets, new vitamins,
and other wonder foods. There are numerous unsubstantiated reports that

54
natural vitamins are superior to synthetic ones, that fertilized eggs are nutritionally
superior to unfertilized eggs, that untreated grains are better than fumigated grains
and the like.
One thing that most organically grown food products seem to have in common is
that they cost more than conventionally grown foods. But in many cases
consumers are misled if they believe organic foods can maintain health and
provide better nutritional quality than conventionally grown foods. So there is real
cause for concern if consumers, particularly those with limited incomes, distrust
the regular food and buy and buy only expensive organic foods instead.
21. The world "Advocates" in paragraph 1 is closest in meaning to which of the following?
A. Proponents
B. Merchants
C. Inspectors
D. Consumers
22. The word "others" in paragraph 1 refers to
A. advantages
B. advocates
C. organic foods
D. products
23. The "welcome development" mentioned in paragraph 2 is an increase in
A. interest in food safety and nutritional quality of the typical North American diet
B. the nutritional quality of the typical North American diet
C. the amount of healthy food grown in North America
D. the number of consumers in North America
24. According to the first paragraph, which of the following is true about the term "organic
foods"?
A. It is accepted by most nutritionists.
B. It has been used only in recent years.
C. It has no fixed meaning.
D. It is seldom used by consumers.
25. The word "unsubstantiated" in paragraph 3 is closest in meaning to
A. unbelievable
B. uncontested
C. unpopular
D. unverified

55
26. The word ‘ones’ in paragraph 3 can best be replaced by
A. vitamins B. reports
C. foods D. diets
27. The word "maintain" in paragraph 4 is closest in meaning to
A. improve B. monitor
C. preserve D. restore
28. The author implies that there is cause for concern if consumers with limited incomes
buy organic foods instead of conventionally grown foods because
A. organic foods can be more expensive but are often no better than conventionally
grown foods.
B. many organic foods are actually less nutritious than similar conventionally grown
foods.
C. conventionally grown foods are more readily available than organic foods.
D. too many farmers will stop using conventional methods to grow food crops.
29. According to the last paragraph, consumers who believe that organic foods are better
than conventionally grown foods are often
A. careless
B. mistaken
C. thrifty
D. wealthy
30. What is the author's attitude toward the claims made by advocates of health foods?
A. Very enthusiastic
B. Somewhat favorable
C. Neutral
D. Skeptical

PASSAGE 4 – Questions 31 – 40
Information technology is influencing the way many of us live and work today. We
use the Internet to look and apply for jobs, shop, conduct research, make airline
reservations, and explore areas of interest. We use e-mail and the Internet to
communicate instantaneously with friends and business associates around the
world. Computers are commonplace in homes and the workplace.
Although the number of Internet users is growing exponentially each year,
most of the world’s population does not have access to computers or the
Internet. Only 6 percent of the population in developing countries is connected to
telephones. Although more than 94 percent of U.S households have a telephone,

56
only 42 percent have personal computers at home and 26 percent have Internet
access. The lack of what most of us would consider a basic communications
necessity- the telephone- does not occur just in developing nations. On some
Native American reservations only 60 percent of the residents have a telephone.
The move to wireless connections may eliminate the need for telephone line, but
it does not remove the barrier to equipment costs.
Who has Internet access? Fifty percent of the children in urban households with
an income over $75,000 have Internet access, compared with 2 percent of the
children in low-income, rural households. Nearly half of college-educated people
have Internet access, compared to 6 percent of those with only some high school
education. Forty percent of households with two parents have access; 15 percent
of female, single-parent households do. Thirty percent of white households, 11
percent of black households, and 13 percent of Hispanic households have access.
Teens and children are the fastest-growing segments of Internet users. The digital
divide between the populations who have access to the Internet and information
technology tools is based on income, race, education, household type, and
geographic location. Only 16 percent of the rural poor, rural and central city
minorities, young householders, and single-parent female households are
connected.
Another problem that exacerbates these disparities is that African-Americans,
Hispanics, and Native Americans hold few of the jobs in information technology.
Women hold about 20 percent of these jobs and are receiving fewer than 30
percent of the computer science degrees. The result is that women and members
of most oppressed ethnic groups are not eligible for the jobs with the highest
salaries at graduation. Baccalaureate candidates with degrees in computer
science were offered the highest salaries of all new college graduates in 1998 at
$44,949.
Do similar disparities exist in schools? More than 90 percent of all schools in the
country are wired with at least one Internet connection. The number of classrooms
with Internet connections differs by the income level of students. Using the
percentage of students who are eligible for free lunches at a school to determine
income level, we see that nearly twice as many of the schools with more affluent
students have wired classrooms as those with high concentrations of low-
income students.
Access to computers and the Internet will be important in reducing disparities
between groups. It will require greater equality across diverse groups whose
members develop knowledge and skills in computer and information technologies.
If computers and the Internet are to be used to promote equality, they will have to
become accessible to populations that cannot currently afford the equipment
which needs to be updated every three years or so. However, access alone is not
enough. Students will have to be interacting with the technology in authentic
settings. As technology becomes a tool for learning in almost all courses taken by
students, it will be seen as a means to an end rather than an end in itself. If it is
used in culturally relevant ways, all students can benefit from its power.

57
31. Why does the author mention the telephone in paragraph 2?
A. to demonstrate that even technology like the telephone is not available to all
B. to argue that basic telephone service is a first step to using the Internet
C. to contrast the absence of telephone usage with that of Internet usage
D. to describe the development of communications from telephone to Internet
32. Which of the sentences below express the essential information in the bold sentence
in paragraph 2? Incorrect choices change the meaning in important ways or leave out
essential information.
A. Most of the people in the world use the Internet now because the number of
computers has been increasing every year.
B. The number of people who use computers and the Internet is increasing every
year, but most people in the world still do not have connections.
C. The number of computers that can make the Internet available to most of the
people in the world is not increasing fast enough.
D. The Internet is available to most of the people in the world, even though they don’t
have their own computer terminals.
33. The word “eliminate” in paragraph 2 is closest meaning to
A. accept B. dispute
C. define D. remove
34. Based on information in paragraph 3, which of the following best explains the term
“digital divide” in paragraph 3?
A. The number of the Internet users in developing nations
B. The disparity in the opportunity to use the Internet
C. Differences in socioeconomic levels among Internet users
D. Segments of the population with Internet access
35. Why does the author give details about the percentages of Internet users in paragraph
3?
A. To prove that there are differences in opportunities among social groups
B. To argue for more Internet connections at all levels of society
C. To suggest that improvements in Internet access are beginning to take place
D. To explain why many people have Internet connections now
36. According to paragraph 3, which of the following households would be least likely to
have access to the Internet?
A. A household with one parent
B.A black household

58
C. A Hispanic household
D.A household with both parents
37. The word “those” in paragraph 5 refers to
A. classrooms
B. students
C. schools
D. concentrations
38. According to paragraph 4, why are fewer women and minorities employed in the field
of computer technology?
A. They are not admitted to the degree programs.
B. They do not possess the educational qualifications.
C. They do not have an interest in technology.
D. They prefer training for jobs with higher salaries.
39. The word “concentrations” in paragraph 5 is closest in meaning to
A. protections
B. numbers
C. confidence
D. support
40. What can be inferred from paragraph 6 about Internet access?
A. Better computers need to be designed.
B. Schools should provide newer computers for students.
C. The cost of replacing equipment is a problem.
D. Technology will be more helpful in three years.

59
PRACTICE TEST 5

PASSAGE 1 – Questions 1 – 10
A rather surprising geographical feature of Antarctica is that a huge freshwater lake,
one of the world’s largest and deepest, lies hidden there under four kilometers of
ice. Now known as Lake Vostok, this huge body of water is located under the ice
block that comprises Antarctica. The lake is able to exist in its unfrozen state beneath
this block of ice because its waters are warmed by geothermal heat from the earth’s
core. The thick glacier above Lake Vostok actually insulates it from the frigid
temperatures (the lowest ever recorded on Earth) on the surface.
The lake was first discovered in the 1970s while a research team was conducting an
aerial survey of the area. Radio waves from the survey equipment penetrated the
ice and revealed a body of water of indeterminate size. It was not until much more
recently that data collected by satellite made scientists aware of the tremendous size
of the lake; the satellite-borne radar detected an extremely flat region where the ice
remains level because it is floating on the water of the lake.
The discovery of such a huge freshwater lake trapped under Antarctica is of interest
to the scientific community because of the potential that the lake contains ancient
microbes that have survived for thousands upon thousands of years, unaffected by
factors such as nuclear fallout and elevated ultraviolet light that have affected
organisms in more exposed areas. The downside of the discovery; however, lies in
the difficulty of conducting research on the lake in such a harsh climate and in the
problems associated with obtaining uncontaminated samples from the lake without
actually exposing the lake to contamination. Scientists are looking for possible ways
to accomplish this.
1. The purpose of the passage is to
A. explain how Lake Vostok was discovered
B. provide satellite data concerning Antarctica
C. discuss future plans for Lake Vostok
D. present an unexpected aspect of Antarctica’s geography
2. The word “lies” in paragraph 1 could best be replaced by
A. sleeps B. sits
C. tells falsehoods D. inclines
3. What is true of Lake Vostok?
A. It is completely frozen.
B. It is a saltwater lake.
C. It is beneath a thick slab of ice.
D. It is heated by the sun.

60
4. Which of the following is closest in meaning to “frigid” in paragraph 1?
A. Extremely cold B. Never changing
C. Quite harsh D. Rarely recorded
5. All of the following are true about the 1970 survey of Antarctica EXCEPT that it
A. was conducted by air
B. made use of radio waves
C. did not measure the exact size of the lake
D. was controlled by a satellite
6. It can be inferred from the passage that the ice would not be flat if
A. there were no lake
B. the lake were not so big
C. Antarctica were not so cold
D. radio waves were not used
7. The word “microbes” in paragraph 3 could best be replaced by
A. Pieces of dust B. Trapped bubbles
C. Tiny organisms D. Rays of light
8. The passage mentions which of the following as a reason for the importance of Lake
Vostok to scientists
A. It can be studied using radio waves.
B. It may contain uncontaminated microbes.
C. It may have elevated levels of ultraviolet light.
D. It has already been contaminated.
9. The word “downside” in paragraph 3 is closest in meaning to
A. bottom level
B. negative aspect
C. underside
D. buried section
10. The paragraph following the passage most probably discusses
A. further discoveries on the surface of Antarctica
B. problems with satellite-borne radar equipment
C. ways to study Lake Vostok without contaminating it
D. the harsh climate of Antarctica

61
PASSAGE 2 – Questions 11 – 20
In the American colonies there was little money. England did not supply the colonies
with coins and it did not allow the colonies to make their own coins, except for the
Massachusetts Bay Colony, which received permission for a short period in 1652 to
make several kinds of silver coins. England wanted to keep money out of America
as a means of controlling trade: America was forced to trade only with England if it
did not have the money to buy products from other countries. The result during this
pre-revolutionary period was that colonists used various goods in place of money:
beaver pelts, Indian wampum, and tobacco leaves were all commonly used
substitutes for money. The colonists also made use of any foreign coins they could
obtain. Dutch, Spanish, French, and English coins were all in use in the American
colonies.
During the Revolutionary War, funds were needed to finance the war, so each of the
individual states and the Continental Congress issued paper money. So much of this
paper money was printed that, by the end of the war, almost no one would accept
it. As a result trade in goods and the use of foreign coins still flourished during this
period.
By the time the Revolutionary War had been won by the American colonists, the
monetary systems in a state of total disarray. To remedy this situation, the new
Constitution of the United States, approved in 1789, allowed Congress to issue
money. The individual states could no longer have their own money supply. A few
years later, the Coinage Act of 1792 made the dollar the official currency of the
United States and put the country on a bimetallic standard. In this bimetallic system,
both gold and silver were legal money, and the rate of exchange of silver to gold was
fixed by the government at sixteen to one.
11. The passage mainly discusses
A. American money from past to present
B. the English monetary policies in colonial America
C. the effect of the Revolution on American money
D. the American monetary system of the seventeenth and eighteenth centuries
12. The passage indicates that during the colonial period, money was
A. supplied by England B. coined by the colonists
C. scarce D. used extensively for trade
13. The Massachusetts Bay Colony was allowed to make coins
A. continuously from the inception of the colony
B. throughout the seventeenth century
C. from 1652 until the Revolutionary War
D. for a short time during one year

62
14. The expression “a means of” in paragraph 1 could best be replaced by
A. an example of B. a method for
C. a result of D. a punishment for
15. Which of the following is NOT mentioned in the passage as a substitute for money
during the colonial period?
A. Wampum
B. Cotton
C. Tobacco
D. Beaver furs
16. The pronoun “it” in paragraph 2 refers to
A. The Continental Congress
B. Paper money
C. The war
D. Trade in goods
17. The word “remedy” in paragraph 3 is closest in meaning to
A. resolve B. understand
C. renew D. medicate
18. How was the monetary system arranged in the Constitution?
A. Only the United States Congress could issue money.
B. The United States officially went on a bimetallic monetary system.
C. Various state governments, including Massachusetts, could issue money.
D. The dollar was made the official currency of the United States.
19. According to the passage, which of the following is NOT true about the bimetallic
monetary system?
A. Either gold or silver could be used as official money.
B. Gold could be exchanged for silver at a rate of sixteen to one.
C. The monetary system was based on two metals.
D. It was established in 1792.
20. The word “fixed” in paragraph 3 is closest in meaning to
A. discovered
B. repaired
C. valued
D. set

63
PASSAGE 3 – Questions 21 – 30
The human brain, with an average weight of 1.4 kilograms, is the control center of the
body. It receives information from the senses, processes the information, and rapidly
sends out responses; it also stores the information that is the source of human
thoughts and feelings. Each of the three main parts of the brain-the cerebrum, the
cerebellum, and the brain stem-has its own role in carrying out these functions.
The cerebrum is by far the largest of the three parts, taking up 85 percent of the
brain by weight. The outside layer of cerebrum, the cerebral cortex, is a grooved
and bumpy surface covering the nerve cells beneath. The various sections of the
cerebrum are the sensory cortex, which is responsible for receiving and decoding
sensory messages from throughout the body; the motor cortex, which sends action
instructions to the skeletal muscles; and the association cortex, which receives,
monitors, and processes information. It is in the association cortex that the
processes that allow humans to think take place.
The cerebellum, located below the cerebrum in the back part of the skull, is made of
masses of bunched up nerve cells. It is the cerebellum that controls human balance,
coordination, and posture.
The brain stem, which connects the cerebrum and the spinal cord, controls various
body processes such as breathing and heartbeat. It is the major motor and sensory
pathway connecting the body and the cerebrum.

21. What is the author’s main purpose?


A. To describe the functions of the parts of the brain
B. To explain how the brain processes information
C. To demonstrate the physical composition of the brain
D. To give examples of human body functions
22. The word “stores” in paragraph 1 is closest in meaning to
A. shops B. processes
C. releases D. stockpiles
23. The passage states that the most massive part of the brain is the
A. cerebrum B. cerebellum
C. cerebral cortex D. brain stem
24. The “cerebral cortex” in paragraph 2 is
A. a layer of the brain beneath the cerebrum
B. a layer of nerve cells in the brain
C. a part of the brain that makes up 85 percent of the brain
D. a ridged layer covering the cerebrum in the brain

64
25. The sensory cortex
A. senses that messages should be sent out to the muscles
B. provides a surface covering for nerve cells
C. is where the human process of thinking occurs
D. receives and processes information from the senses
26. The word “monitors” in paragraph 2 is closest in meaning to
A. Keeps track of B. Keeps hold of
C. Gets away with D. Gets rid up
27. Which of the following is NOT a part of the cerebrum?
A. cerebral cortex B. sensory cortex
C. association cortex D. cerebellum
28. Which of the following is true about the cerebellum?
A. It is located above the cerebrum.
B. It controls breathing.
C. It is responsible for balance.
D. It is the outside layer of the cerebrum.
29. What shape does the brain stem most likely have?
A. Small and round B. Long and thin
C. Large and formless D. Short and flat
30. Which of the following could best replace the word “pathway” in paragraph 4?
A. Driveway B. Roadway
C. Route D. Street

READING 4 – Questions 31 – 40
The Netherlands, in Europe, is more than 60 percent near or below sea level. Flooding
has always been a major worry because its major cities, factories and green house farms
are built on floodplains. The greatest risk of flooding comes from the North Sea itself with
its severe winter storms and massive tides. However, flooding can also occur along the
Rhine, Maas and Scheldt Rivers, which are swollen every spring with snowmelt from the
Alps. And when these major rivers meet the North Sea, they form a delta region that
consists of additional floodplains.
As the Dutch built their cities, farms and factories on the floodplains, they also build a
system of barriers to keep the floodwaters back. The Dutch barriers are the strongest in
the world, designed to protect the country from the usual high tides and heavy rainfall, as
well as possible “monster” storms that occur only rarely. The first line of defense is the

65
dikes, dams, and storm barriers, which over the centuries have become ever longer and
higher. Today they stretch for about 2,200 miles along the sea and the major rivers. ■ [A]
They are aided by secondary defenses, such as canal dikes and holding ponds, as well
as by thousands of water pumps and pumping stations. ■ [B] A recent government study
on the effects of climate change has led the Dutch government to change its traditional
water protection strategies. ■ [C] At the same time, hotter summers could lead to cracking
and weakening in the traditional earthen dikes. ■ [D]
Thus, in the future, it might no longer be possible to hold back the water. Though it
will continue to maintain the barriers, the government is shifting to a “softer”, more
environmentally friendly approach. The aim is to rely more on the natural protection of
flood plains, sand dunes, salt marshes, and mud flats. The government has begun buying
land along major waterways, where water could be directed when floods threaten.

31. What can be inferred about people living in the Netherlands?


A. They constantly suffer serious damages caused by floods.
B. They build their houses where floods cannot approach.
C. They are used to living with floods.
D. Flood becomes their big concern.
32. Why did the Dutch build a system of barriers?
A. To use floodwater for farming.
B. To protect the countryside from the tides.
C. To prevent the country from being flooded.
D. To keep the country away from strong winds.
33. The word “massive” in paragraph 1 is closest in meaning to
A. low B. minor
C. huge D. predictable
34. What do the Dutch do today to strengthen their system of barriers?
A. They rearrange barriers in one line.
B. They build houses on high lands.
C. They build more water-processing works.
D. They rebuild the existing system.
35. What does the word “they” in paragraph 2 refer to?
A. centuries B. Dikes, dams and barriers
C. the Dutch D. a system of barriers
36. Look at the gaps [■] in the second paragraph. Where does the following sentence
best fit?
The study concluded that the combination of higher rainfall in Northern Europe
and rising sea levels would lead to an increased risk of flooding.
A. [A] B. [B] C. [C] D. [D]

66
37. Which of the following best paraphrases the first sentence of the third paragraph?
A. The Netherlands should give up fighting against floods.
B. The Dutch will probably not be able to hold back the water.
C. In the future, the Dutch houses will be under water.
D. In the future, it will be impossible to use the water.
38. Which of the following is the Dutch government NOT going to do?
A. Maintaining their system of barriers
B. Shifting to an environmentally-friendly approach
C. Finding ways to direct the floodwaters
D. Replacing its existing system of dikes and lakes
39. What can be inferred about the Dutch government?
A. It is going to invest more in waterways.
B. It has invested in water transport.
C. It has made great efforts to recycle flood water.
D. It has placed a stress on protecting the country from floods.
40. Where is the passage most probably found?
A. In a comic book
B. In a novel
C. In a bibliography
D. In a geographical document

67

You might also like